Уравнения логарифмов: Логарифмические уравнения и неравенства — Умскул Учебник

Содержание

Логарифмические уравнения на примерах

Логарифмическими называются уравнения содержащие неизвестную величину под знаком логарифма или в основании логарифма (или в обоих местах одновременно). Их легко свести к квадратным или степенным уравнениям относительно переменной если знать свойства логарифма. Например, логарифмическими будут следующие уравнения


Необходимо отметить что во время решения логарифмических уравнений необходимо учитывать область допустимых значений ( ОДЗ ) : под знаком логарифма могут находиться только положительные величины, в основе логарифмов — положительные, отличные от единицы. Однако нахождения ОДЗ порой может быть очень громоздким и на практике имеем возможность или искать ОДЗ, или сделать проверку подстановкой корней уравнения.

Простейшим логарифмическим уравнением называют уравнение вида

Его решение вычисляется потенцированием (нахождение числа или выражения по его логарифму)

В некоторых случаях, решая логарифмические уравнения, целесообразно производить замену переменной. Например в уравнении

удобно сделать замену и мы приходим к квадратному уравнению. Причем оба корни этого квадратного уравнения можно подставить в замену чтобы найти подходящее х.

Стоит запомнить что десятичный логарифм от единицы со следующими нулями равно количеству нулей в записи этого числа.

Для десятичного логарифма от единицы с предыдущими нулями правило подобное. Он равен количеству всех нулей в записи этого числа, включая и ноль целых, взятых со знаком минус. Для примера

На этом необходимый теоретический материал рассмотрен и можно переходить к рассмотрению практических примеров. Внимательно рассмотрите их решения это позволит усвоить некоторые правила логарифмов и увеличит практическую базу, которая пригодится при прохождении ВНО , контрольных, тестах и т.д.

Пример 1. Решить уравнение.

Решение. Используя свойство логарифмов переписываем уравнение в виде

Делаем замену

и переписываем

Умножаем на переменную и записываем в виде квадратного уравнения

Вычисляем дискриминант

Корни уравнения приобретут значения

Возвращаемся к замене и находим


Уравнение имеет два решения

 

Пример 2. Решить уравнение.

Решение. Раскрываем скобки и записываем в виде суммы логарифмов

Учитывая что уравнение примет вид

Переносим слагаемое за знаком равенства в правую сторону


Оба множители приравниваем к нулю и находим

 

Пример 3. Решить уравнение.

Решение. Перепишем правую сторону в виде квадрата и прологарифмируем по основанию 10 обе части уравнения

делаем замену

и сводим уравнение к квадратному

Дискриминант такого уравнения принимает нулевое значение — уравнение имеет два одинаковых решения

Возвращаемся к замене которую делали выше

 

Пример 4. Решить уравнение.

Решение. Выполним некоторые преобразования с слагаемыми уравнения



Логарифмическое уравнение упростится до следующего

Поскольку логарифмы имеют одинаковые основания то значение под знаком логарифма тоже равны. На основе этого имеем

Расписываем и решаем с помощью дискриминанта



Второй корень не может быть решением, поскольку никакое положительное число при возведены в степени не даст в результате -1. Итак x=2 – единственное решение уравнения.

 

Пример 5. Найти решение уравнения .

Решение. Выполняем упрощения уравнения




По свойству переходим ко второй основы во втором логарифме



По правилу логарифмирования имеем

Сводим уравнение к квадратному и решаем его


Дискриминант равен нулю, следовательно имеем один корень кратности два

 

Пример 6. Найти решение уравнения.

Решение. Заданное уравнение и подобные ему решаются путем сведения к общей основе. Для этого преобразуем правую сторону уравнения к виду

и подставим в уравнение

Поскольку основы логарифмов ровны переходим до показательного уравнения

Выполняем замену и сводим к квадратному уравнению



Возвращаемся к замене и вычисляем

 

Пример 7. Найти решение уравнения.

Решение. Не пугайтесь подобных задач, если делать все по правилам то решение получается без труда. Забегая вперед скажу что корни в скобках к примеру отношения не имеют. Они для того чтобы напугать простых математиков.
Упростим сначала второй логарифм

Дальше выполняем подстановку и сведения слагаемых под один логарифм

Приравниваем к правой части уравнения и упрощаем




Как видите — решение оказалось проще чем выглядело до решения, а результат x=100 только подтверждает это.

При решении логарифмических уравнений важно хорошо знать свойства логарифмов. Все остальные действия сводятся, как правило, к решению квадратных уравнений или степенных зависимостей относительно неизвестных. Поэтому практикуйте самостоятельно и не имейте проблем с логарифмическими уравнениями.

Урок 4. Логарифмическая функция. Логарифмические уравнения. Системы логарифмических уравнений. Теория. 11 класс

 

 

Подготовка к ЕГЭ по математике

 

 

Эксперимент

 

Урок 4. Логарифмическая функция. Логарифмические уравнения. Системы логарифмических уравнений.

 

Теория

 

Конспект урока

На предыдущем уроке мы определили понятие логарифма, обсудили его основные свойства.

Сегодня мы поговорим о решении простейших логарифмических уравнений и видах логарифмических уравнений.

По аналогии с решением показательных уравнений мы воспользуемся свойствами логарифмической функции для решения логарифмических уравнений.

 

Логарифмическая функция

 

 

Рассмотрим логарифмическую функцию:

 

Рассмотрим её свойства:

1)   – это следует из определения логарифма (под логарифмом не может стоять отрицательное число или 0)

2)    Стоит отметить, что показательная и логарифмическая функции являются взаимно обратными. Поэтому область определения показательной функции совпадает с областью значения логарифмической и наоборот (более подробно о свойствах прямой и обратной функции мы поговорим в теме «Функции»).

3)  Точки пересечения с осями.

Ох (нули функции):  так как логарифм от 1 по любому основанию равен 0 (любое положительное число в 0 степени равно 1). Значит, график логарифмической функции проходит через точку

Оу:  – не существует, так как 0 не входит в область определения логарифмической функции.

4)   Функция не является ни чётной, ни нечётной (функция общего вида), так как область определения не симметрична относительно 0 (то есть, функция не определена при отрицательных значениях переменной).

Также функция не является периодической.

5)   При  функция монотонно возрастает на всей области определения (обратите внимание на сходство с показательной функцией).

При  функция монотонно убывает на всей области определения.

6)    Графики логарифмической функции при  и имеют вид:

 

Схема решения логарифмических уравнений

 

 

Мы видим, что логарифмическая функция, как и показательная, является монотонной (монотонно возрастает при  и монотонно убывает при ).

 

Это означает, что мы можем по аналогии с простейшими показательными уравнениями определить способ решения простейших логарифмических уравнений ():  Однако при этом необходимо помнить, что под логарифмом должно стоять положительное число. Таким образом, при решении логарифмических уравнений необходимо учитывать ОДЗ, а именно: проверять, что все подлогарифмические выражения, а также основания логарифмов являются положительными и основания не равны 1.

Однако можно избежать определения ОДЗ исходного уравнения, выполнив в конце проверку полученных результатов (поскольку мы не сужаем область поиска корней, а расширяем её). В большинстве случаев такой подход облегчает решение логарифмических уравнений.

Таким образом, для решения простейшего логарифмического уравнения достаточно привести обе части к одинаковому основанию, а затем приравнять подлогарифмические выражения.

Например:

Правда, в данном конкретном случае мы могли воспользоваться и определением логарифма:  Однако продемонстрированный метод более универсальный.

Любое более сложное логарифмическое уравнение решается «выливанием воды из чайника», то есть сведением его различными методами к простейшим.

 

 

Виды логарифмических уравнений

 

 

1) Простейшие

 

2)  Простейшие с переменной в основании логарифма

3)  Простейшие с переменной и в основании, и под логарифмом

4)  Сводящиеся к простейшим с помощью использования свойств логарифмов

5)  Сводящиеся к квадратным

 

Системы логарифмических уравнений

 

 

Системы логарифмических уравнений решаются по тем же принципам, что и системы показательных уравнений.

 

Самые простые системы логарифмических уравнений – это системы, в которых оба уравнения сводятся к простейшим. В дальнейшем получается обычная система из двух уравнений с двумя неизвестными, которая решается любым из удобных методов.

Пример такой системы: .

Ещё один важный тип систем логарифмических уравнений – это системы, которые сводятся к обычным с помощью замены. Пример такой системы: .

Также существуют системы логарифмических уравнений, которые решаются различными методами.

Более подробно о решении систем логарифмических уравнений мы поговорим в практической части урока.

На этом уроке мы с вами обсудили свойства логарифмической функции, научились решать простейшие логарифмические уравнения. Также мы узнали об основных видах логарифмических уравнений и их систем.

В практической части урока мы научимся решать различные логарифмические уравнения и их системы.

 

Полезные ссылки:

1)      Алгебра 11 класс: «Функция y=logax, ее свойства и график» 

2)      Алгебра 11 класс: «Функция y=logax, ее свойства и график (продолжение)» 

3)      Алгебра 11 класс: «Функция y=logax, ее свойства и график. Решение задач» 

4)      Алгебра 11 класс: «Логарифмические уравнения» 

5)      Алгебра 11 класс: «Решение логарифмических уравнений» 

6)      Алгебра 11 класс: «Решение логарифмических уравнений» 

 

Логарифмические уравнения и методы их решения.

Решение логарифмических уравнений

Главная > Двери > Логарифмические уравнения и методы их решения. Решение логарифмических уравнений

Как известно, при перемножении выражений со степенями их показатели всегда складываются (a b *a c = a b+c). Этот математический закон был выведен Архимедом, а позже, в VIII веке, математик Вирасен создал таблицу целых показателей. Именно они послужили для дальнейшего открытия логарифмов. Примеры использования этой функции можно встретить практически везде, где требуется упростить громоздкое умножение на простое сложение. Если вы потратите минут 10 на прочтение этой статьи, мы вам объясним, что такое логарифмы и как с ними работать. Простым и доступным языком.

Определение в математике

Логарифмом называется выражение следующего вида: log a b=c, то есть логарифмом любого неотрицательного числа (то есть любого положительного) «b» по его основанию «a» считается степень «c», в которую необходимо возвести основание «a», чтобы в итоге получить значение «b». Разберем логарифм на примерах, допустим, есть выражение log 2 8. Как найти ответ? Очень просто, нужно найти такую степень, чтобы из 2 в искомой степени получить 8. Проделав в уме некоторые расчеты, получаем число 3! И верно, ведь 2 в степени 3 дает в ответе число 8.

Разновидности логарифмов

Для многих учеников и студентов эта тема кажется сложной и непонятной, однако на самом деле логарифмы не так страшны, главное — понять общий их смысл и запомнить их свойста и некоторые правила. Существует три отдельных вида логарифмических выражений:

  1. Натуральный логарифм ln a, где основанием является число Эйлера (e = 2,7).
  2. Десятичный a, где основанием служит число 10.
  3. Логарифм любого числа b по основанию a>1.

Каждый из них решается стандартным способом, включающим в себя упрощение, сокращение и последующее приведение к одному логарифму с помощью логарифмических теорем. Для получения верных значений логарифмов следует запомнить их свойства и очередность действий при их решениях.

Правила и некоторые ограничения

В математике существует несколько правил-ограничений, которые принимаются как аксиома, то есть не подлежат обсуждению и являются истиной. Например, нельзя числа делить на ноль, а еще невозможно извлечь корень четной степени из отрицательных чисел. Логарифмы также имеют свои правила, следуя которым можно с легкостью научиться работать даже с длинными и емкими логарифмическими выражениями:

  • основание «a» всегда должно быть больше нуля, и при этом не быть равным 1, иначе выражение потеряет свой смысл, ведь «1» и «0» в любой степени всегда равны своим значениям;
  • если а > 0, то и а b >0, получается, что и «с» должно быть больше нуля.

Как решать логарифмы?

К примеру, дано задание найти ответ уравнения 10 х = 100. Это очень легко, нужно подобрать такую степень, возведя в которую число десять, мы получим 100. Это, конечно же, 10 2 =100.

А теперь давайте представим данное выражение в виде логарифмического. Получим log 10 100 = 2. При решении логарифмов все действия практически сходятся к тому, чтобы найти ту степень, в которую необходимо ввести основание логарифма, чтобы получить заданное число.

Для безошибочного определения значенияя неизвестной степени необходимо научиться работать с таблицей степеней. Выглядит она следующим образом:

Как видите, некоторые показатели степени можно угадать интуитивно, если имеется технический склад ума и знание таблицы умножения. Однако для больших значений потребуется таблица степеней. Ею могут пользоваться даже те, кто совсем ничего не смыслит в сложных математических темах. В левом столбце указаны числа (основание a), верхний ряд чисел — это значение степени c, в которую возводится число a. На пересечении в ячейках определены значения чисел, являющиеся ответом (a c =b). Возьмем, к примеру, самую первую ячейку с числом 10 и возведем ее в квадрат, получим значение 100, которое указано на пересечении двух наших ячеек. Все так просто и легко, что поймет даже самый настоящий гуманитарий!

Уравнения и неравенства

Получается, что при определенных условиях показатель степени — это и есть логарифм. Следовательно, любые математические численные выражения можно записать в виде логарифмического равенства. Например, 3 4 =81 можно записать в виде логарифма числа 81 по основанию 3, равному четырем (log 3 81 = 4). Для отрицательных степеней правила такие же: 2 -5 = 1/32 запишем в виде логарифма, получим log 2 (1/32) = -5. Одной из самых увлекательных разделов математики является тема «логарифмы». Примеры и решения уравнений мы рассмотрим чуть ниже, сразу же после изучения их свойств. А сейчас давайте разберем, как выглядят неравенства и как их отличить от уравнений.

Дано выражение следующего вида: log 2 (x-1) > 3 — оно является логарифмическим неравенством, так как неизвестное значение «х» находится под знаком логарифма. А также в выражении сравниваются две величины: логарифм искомого числа по основанию два больше, чем число три.

Самое главное отличие между логарифмическими уравнениями и неравенствами заключается в том, что уравнения с логарифмами (пример — логарифм 2 x = √9) подразумевают в ответе одно или несколько определенных числовых значений, тогда как при решении неравенства определяются как область допустимых значений, так и точки разрыва этой функции. Как следствие, в ответе получается не простое множество отдельных чисел как в ответе уравнения, а а непрерывный ряд или набор чисел.

Основные теоремы о логарифмах

При решении примитивных заданий по нахождению значений логарифма, его свойства можно и не знать. Однако когда речь заходит о логарифмических уравнениях или неравенствах, в первую очередь, необходимо четко понимать и применять на практике все основные свойства логарифмов. С примерами уравнений мы познакомимся позже, давайте сначала разберем каждое свойство более подробно.

  1. Основное тождество выглядит так: а logaB =B. Оно применяется только при условии, когда а больше 0, не равно единице и B больше нуля.
  2. Логарифм произведения можно представить в следующей формуле: log d (s 1 *s 2) = log d s 1 + log d s 2. При этом обязательным условием является: d, s 1 и s 2 > 0; а≠1. Можно привести доказательство для этой формулы логарифмов, с примерами и решением. Пусть log a s 1 = f 1 и log a s 2 = f 2 , тогда a f1 = s 1 , a f2 = s 2. Получаем, что s 1 *s 2 = a f1 *a f2 = a f1+f2 (свойства степеней), а далее по определению: log a (s 1 *s 2)= f 1 + f 2 = log a s1 + log a s 2, что и требовалось доказать.
  3. Логарифм частного выглядит так: log a (s 1/ s 2) = log a s 1 — log a s 2.
  4. Теорема в виде формулы приобретает следующий вид: log a q b n = n/q log a b.

Называется эта формула «свойством степени логарифма». Она напоминает собой свойства обычных степеней, и неудивительно, ведь вся математика держится на закономерных постулатах. Давайте посмотрим на доказательство.

Пусть log a b = t, получается a t =b. Если возвести обе части в степень m: a tn = b n ;

но так как a tn = (a q) nt/q = b n , следовательно log a q b n = (n*t)/t, тогда log a q b n = n/q log a b. Теорема доказана.

Примеры задач и неравенств

Самые распространенные типы задач на тему логарифмов — примеры уравнений и неравенств. Они встречаются практически во всех задачниках, а также входят в обязательную часть экзаменов по математике. Для поступления в университет или сдачи вступительных испытаний по математике необходимо знать, как правильно решать подобные задания.

К сожалению, единого плана или схемы по решению и определению неизвестного значения логарифма не существует, однако к каждому математическому неравенству или логарифмическому уравнению можно применить определенные правила. Прежде всего следует выяснить, можно ли упростить выражение или привести к общему виду. Упрощать длинные логарифмические выражения можно, если правильно использовать их свойства. Давайте скорее с ними познакомимся.

При решении же логарифмических уравнений, следует определить, какой перед нами вид логарифма: пример выражения может содержать натуральный логарифм или же десятичный.

Вот примеры ln100, ln1026. Их решение сводится к тому, что нужно определить ту степень, в которой основание 10 будет равно 100 и 1026 соответственно. Для решений же натуральных логарифмов нужно применить логарифмические тождества или же их свойства. Давайте на примерах рассмотрим решение логарифмических задач разного типа.

Как использовать формулы логарифмов: с примерами и решениями

Итак, рассмотрим примеры использования основных теорем о логарифмах.

  1. Свойство логарифма произведения можно применять в заданиях, где необходимо разложить большое значение числа b на более простые сомножители. Например, log 2 4 + log 2 128 = log 2 (4*128) = log 2 512. Ответ равен 9.
  2. log 4 8 = log 2 2 2 3 = 3/2 log 2 2 = 1,5 — как видите, применяя четвертое свойство степени логарифма, удалось решить на первый взгляд сложное и нерешаемое выражение. Необходимо всего лишь разложить основание на множители и затем вынести значения степени из знака логарифма.

Задания из ЕГЭ

Логарифмы часто встречаются на вступительных экзаменах, особенно много логарифмических задач в ЕГЭ (государственный экзамен для всех выпускников школ). Обычно эти задания присутствуют не только в части А (самая легкая тестовая часть экзамена), но и в части С (самые сложные и объемные задания).

Экзамен подразумевает точное и идеальное знание темы «Натуральные логарифмы».

Примеры и решения задач взяты из официальных вариантов ЕГЭ. Давайте посмотрим, как решаются такие задания.

Дано log 2 (2x-1) = 4. Решение:
перепишем выражение, немного его упростив log 2 (2x-1) = 2 2 , по определению логарифма получим, что 2x-1 = 2 4 , следовательно 2x = 17; x = 8,5.

  • Все логарифмы лучше всего приводить к одному основанию, чтобы решение не было громоздким и запутанным.
  • Все выражение, стоящие под знаком логарифма, указываются как положительные, поэтому при вынесении множителем показателя степени выражения, который стоит под знаком логарифма и в качестве его основания, остающееся под логарифмом выражение должно быть положительно.

Логарифмические уравнения. От простого — к сложному.

Внимание!
К этой теме имеются дополнительные
материалы в Особом разделе 555.
Для тех, кто сильно «не очень…»
И для тех, кто «очень даже.. .»)

Что такое логарифмическое уравнение?

Это уравнение с логарифмами. Вот удивил, да?) Тогда уточню. Это уравнение, в котором неизвестные (иксы) и выражения с ними находятся внутри логарифмов. И только там! Это важно.

Вот вам примеры логарифмических уравнений :

log 3 х = log 3 9

log 3 (х 2 -3) = log 3 (2х)

log х+1 (х 2 +3х-7) = 2

lg 2 (x+1)+10 = 11lg(x+1)

Ну, вы поняли… )

Обратите внимание! Самые разнообразные выражения с иксами располагаются исключительно внутри логарифмов. Если, вдруг, в уравнении обнаружится икс где-нибудь снаружи , например:

log 2 х = 3+х,

это будет уже уравнение смешанного типа. Такие уравнения не имеют чётких правил решения. Мы их пока рассматривать не будем. Кстати, попадаются уравнения, где внутри логарифмов только числа . Например:

Что тут сказать? Повезло вам, если попалось такое! Логарифм с числами — это

какое-то число. И всё. Достаточно знать свойства логарифмов, чтобы решить такое уравнение. Знания специальных правил, приёмов, приспособленных именно для решения логарифмических уравнений, здесь не требуется.

Итак, что такое логарифмическое уравнение — разобрались.

Как решать логарифмические уравнения?

Решение логарифмических уравнений — штука, вообще-то, не очень простая. Так и раздел у нас — на четвёрку… Требуется приличный запас знаний по всяким смежным темам. Кроме того, существует в этих уравнениях особая фишка. И фишка это настолько важная, что её смело можно назвать главной проблемой в решении логарифмических уравнений. Мы с этой проблемой в следующем уроке детально разберёмся.

А сейчас — не волнуйтесь. Мы пойдём правильным путём, от простого к сложному. На конкретных примерах. Главное, вникайте в простые вещи и не ленитесь ходить по ссылкам, я их не просто так поставил… И всё у вас получится. Обязательно.

Начнём с самых элементарных, простейших уравнений. Для их решения желательно иметь представление о логарифме, но не более того. Просто без понятия логарифма, браться за решение логарифмических уравнений — как-то и неловко даже… Очень смело, я бы сказал).

Простейшие логарифмические уравнения.

Это уравнения вида:

1. log 3 х = log 3 9

2. log 7 (2х-3) = log 7 х

3. log 7 (50х-1) = 2

Процесс решения любого логарифмического уравнения заключается в переходе от уравнения с логарифмами к уравнению без них. В простейших уравнениях этот переход осуществляется в один шаг. Потому и простейшие.)

И решаются такие логарифмические уравнения на удивление просто. Смотрите сами.

Решаем первый пример:

log 3 х = log 3 9

Для решения этого примера почти ничего знать и не надо, да… Чисто интуиция!) Что нам особо не нравится в этом примере? Что-что… Логарифмы не нравятся! Правильно. Вот и избавимся от них. Пристально смотрим на пример, и у нас возникает естественное желание… Прямо-таки непреодолимое! Взять и выкинуть логарифмы вообще. И, что радует, это можно сделать! Математика позволяет. Логарифмы исчезают, получается ответ:

Здорово, правда? Так можно (и нужно) делать всегда. Ликвидация логарифмов подобным образом — один из основных способов решения логарифмических уравнений и неравенств. В математике эта операция называется потенцирование. Есть, конечно, свои правила на такую ликвидацию, но их мало. Запоминаем:

Ликвидировать логарифмы безо всяких опасений можно, если у них:

а) одинаковые числовые основания

в) логарифмы слева-справа чистые (безо всяких коэффициентов) и находятся в гордом одиночестве.

Поясню последний пункт. В уравнении, скажем,

log 3 х = 2log 3 (3х-1)

убирать логарифмы нельзя. Двойка справа не позволяет. Коэффициент, понимаешь… В примере

log 3 х+log 3 (х+1) = log 3 (3+х)

тоже нельзя потенцировать уравнение. В левой части нет одинокого логарифма. Их там два.

Короче, убирать логарифмы можно, если уравнение выглядит так и только так:

log а (. ….) = log а (…..)

В скобках, где многоточие, могут быть какие угодно выражения. Простые, суперсложные, всякие. Какие угодно. Важно то, что после ликвидации логарифмов у нас остаётся более простое уравнение. Предполагается, конечно, что решать линейные, квадратные, дробные, показательные и прочие уравнения без логарифмов вы уже умеете.)

Теперь легко можно решить второй пример:

log 7 (2х-3) = log 7 х

Собственно, в уме решается. Потенцируем, получаем:

Ну что, очень сложно?) Как видите, логарифмическая часть решения уравнения заключается только в ликвидации логарифмов… А дальше идёт решение оставшегося уравнения уже без них. Пустяшное дело.

Решаем третий пример:

log 7 (50х-1) = 2

Видим, что слева стоит логарифм:

Вспоминаем, что этот логарифм — какое-то число, в которое надо возвести основание (т.е. семь), чтобы получить подлогарифменное выражение, т.е. (50х-1).

Но это число равно двум! По уравнению. Стало быть:

Вот, в сущности, и всё. Логарифм исчез, осталось безобидное уравнение:

Мы решили это логарифмическое уравнение исходя только из смысла логарифма. Что, ликвидировать логарифмы всё-таки проще?) Согласен. Между прочим, если из двойки логарифм сделать, можно этот пример и через ликвидацию решить. Из любого числа можно логарифм сделать. Причём, такой, какой нам надо. Очень полезный приём в решении логарифмических уравнений и (особо!) неравенств.

Не умеете из числа логарифм делать!? Ничего страшного. В разделе 555 этот приём подробно описан. Можете освоить и применять его на полную катушку! Он здорово уменьшает количество ошибок.

Совершенно аналогично (по определению) решается и четвёртое уравнение:

Вот и все дела.

Подведём итоги этого урока. Мы рассмотрели на примерах решение простейших логарифмических уравнений. Это очень важно. И не только потому, что такие уравнения бывают на контрольных-экзаменах. Дело в том, что даже самые злые и замороченные уравнения обязательно сводятся к простейшим!

Собственно, простейшие уравнения — это финишная часть решения любых уравнений. И эту финишную часть надо понимать железно! И ещё. Обязательно дочитайте эту страничку до конца. Есть там сюрприз…)

Решаем теперь самостоятельно. Набиваем руку, так сказать…)

Найти корень (или сумму корней, если их несколько) уравнений:

ln(7х+2) = ln(5х+20)

log 2 (х 2 +32) = log 2 (12x)

log 16 (0,5х-1,5) = 0,25

log 0,2 (3х-1) = -3

ln(е 2 +2х-3) = 2

log 2 (14х) = log 2 7 + 2

Ответы (в беспорядке, разумеется): 42; 12; 9; 25; 7; 1,5; 2; 16.

Что, не всё получается? Бывает. Не горюйте! В разделе 555 решение всех этих примеров расписано понятно и подробно. Там уж точно разберётесь. Да ещё и полезные практические приёмы освоите.

Всё получилось!? Все примеры «одной левой»?) Поздравляю!

Пришло время открыть вам горькую правду. Успешное решение этих примеров вовсе не гарантирует успех в решении всех остальных логарифмических уравнений. Даже простейших, подобных этим. Увы.

Дело в том, что решение любого логарифмического уравнения (даже самого элементарного!) состоит из двух равноценных частей. Решение уравнения, и работа с ОДЗ. Одну часть — решение самого уравнения — мы освоили. Не так уж и трудно, верно?

Для этого урока я специально подобрал такие примеры, в которых ОДЗ никак на ответе не сказывается. Но не все такие добрые, как я, правда?…)

Посему надо обязательно освоить и другую часть. ОДЗ. Это и есть главная проблема в решении логарифмических уравнений. И не потому, что трудная — эта часть ещё проще первой. А потому, что про ОДЗ просто забывают. Или не знают. Или и то, и другое). И падают на ровном месте…

В следующем уроке мы расправимся с этой проблемой. Вот тогда можно будет уверенно решать любые несложные логарифмические уравнения и подбираться к вполне солидным заданиям.

Если Вам нравится этот сайт…

Кстати, у меня есть ещё парочка интересных сайтов для Вас.)

Можно потренироваться в решении примеров и узнать свой уровень. Тестирование с мгновенной проверкой. Учимся — с интересом!)

можно познакомиться с функциями и производными.

Решение логарифмических уравнений. Часть 1.

Логарифмическим уравнением называется уравнение, в котором неизвестное содержится под знаком логарифма (в частности, в основании логарифма).

Простейшее логарифмическое уравнение имеет вид:

Решение любого логарифмического уравнения предполагает переход от логарифмов к выражениям, стоящим под знаком логарифмов. Однако это действие расширяет область допустимых значений уравнения и может привести к появлению посторонних корней. Чтобы избежать появления посторонних корней , можно поступить одним из трех способов:

1. Сделать равносильный переход от исходного уравнения к системе, включающей

в зависимости от того, какое неравенство или проще.

Если уравнение содержит неизвестное в основании логарифма:

то мы переходим к системе:

2. Отдельно найти область допустимых значений уравнения , затем решить уравнение и проверить, удовлетворяют ли найденные решения уравнения.

3. Решить уравнение, и потом сделать проверку: подставить найденные решения в исходное уравнение, и проверить, получим ли мы верное равенство.

Логарифмическое уравнение любого уровня сложности в конечном итоге всегда сводится к простейшему логарифмическому уравнению.

Все логарифмические уравнения можно условно разделить на четыре типа:

1 . Уравнения, которые содержат логарифмы только в первой степени. Они с помощью преобразований и использования приводятся к виду

Пример . Решим уравнение:

Приравняем выражения, стоящие под знаком логарифма:

Проверим, удовлетворяет ли наш корень уравнения:

Да, удовлетворяет.

Ответ: х=5

2 . Уравнения, которые содержат логарифмы в степени, отличной от 1 (в частности, в знаменателе дроби). Такие уравнения решаются с помощью введения замены переменной .

Пример. Решим уравнение:

Найдем ОДЗ уравнения:

Уравнение содержит логарифмы в квадрате, поэтому решается с помощью замены переменной.

Важно! Прежде чем вводить замену, нужно «растащить» логарифмы, входящие в состав уравнения на «кирпичики», используя свойства логарифмов.

При «растаскивании» логарифмов важно очень аккуратно применять свойства логарифмов:

Кроме того, здесь есть еще одно тонкое место, и, чтобы избежать распространенной ошибки, воспользуемся промежуточным равенством: запишем степень логарифма в таком виде:

Аналогично,

Подставим полученные выражения в исходное уравнение. Получим:

Теперь мы видим, что неизвестное содержится в уравнении в составе . Введем замену : . Так как может принимать любое действительное значение, на переменную мы никаких ограничений не накладываем.

Логарифмические уравнения. Продолжаем рассматривать задачи из части В ЕГЭ по математике. Мы с вами уже рассмотрели решения некоторых уравнений в статьях « » , « » . В этой статье рассмотрим логарифмические уравнения. Сразу скажу, что никаких сложных преобразований при решении таких уравнений на ЕГЭ не будет. Они просты.

Достаточно знать и понимать основное логарифмическое тождество, знать свойства логарифма. Обратите внимание на то, то после решения ОБЯЗАТЕЛЬНО нужно сделать проверку — подставить полученное значение в исходное уравнение и вычислить, в итоге должно получиться верное равенство.

Определение :

Логарифмом числа a по основанию b называется показатель степени, в который нужно возвести b, чтобы получить a.


Например:

Log 3 9 = 2, так как 3 2 = 9

Свойства логарифмов:

Частные случаи логарифмов:

Решим задачи. В первом примере мы сделаем проверку. В последующих проверку сделайте самостоятельно.

Найдите корень уравнения: log 3 (4–x) = 4

Так как log b a = x b x = a, то

3 4 = 4 – x

x = 4 – 81

x = – 77

Проверка:

log 3 (4–(–77)) = 4

log 3 81 = 4

3 4 = 81 Верно.

Ответ: – 77

Решите самостоятельно:

Найдите корень уравнения: log 2 (4 – x) = 7

Найдите корень уравнения log 5 (4 + x) = 2

Используем основное логарифмическое тождество.

Так как log a b = x b x = a, то

5 2 = 4 + x

x =5 2 – 4

x = 21

Проверка:

log 5 (4 + 21) = 2

log 5 25 = 2

5 2 = 25 Верно.

Ответ: 21

Найдите корень уравнения log 3 (14 – x) = log 3 5.

Имеет место следующее свойство, смысл его таков: если в левой и правой частях уравнения имеем логарифмы с одинаковым основанием, то можем приравнять выражения, стоящие под знаками логарифмов.

14 – x = 5

x = 9

Сделайте проверку.

Ответ: 9

Решите самостоятельно:

Найдите корень уравнения log 5 (5 – x) = log 5 3.

Найдите корень уравнения: log 4 (x + 3) = log 4 (4x – 15).

Если log c a = log c b, то a = b

x + 3 = 4x – 15

3x = 18

x = 6

Сделайте проверку.

Ответ: 6

Найдите корень уравнения log 1/8 (13 – x) = – 2.

(1/8) –2 = 13 – x

8 2 = 13 – x

x = 13 – 64

x = – 51

Сделайте проверку.

Небольшое дополнение – здесь используется свойство

степени ().

Ответ: – 51

Решите самостоятельно:

Найдите корень уравнения: log 1/7 (7 – x) = – 2

Найдите корень уравнения log 2 (4 – x) = 2 log 2 5.

Преобразуем правую часть. воспользуемся свойством:

log a b m = m∙log a b

log 2 (4 – x) = log 2 5 2

Если log c a = log c b, то a = b

4 – x = 5 2

4 – x = 25

x = – 21

Сделайте проверку.

Ответ: – 21

Решите самостоятельно:

Найдите корень уравнения: log 5 (5 – x) = 2 log 5 3

Решите уравнение log 5 (x 2 + 4x) = log 5 (x 2 + 11)

Если log c a = log c b, то a = b

x 2 + 4x = x 2 + 11

4x = 11

x = 2,75

Сделайте проверку.

Ответ: 2,75

Решите самостоятельно:

Найдите корень уравнения log 5 (x 2 + x) = log 5 (x 2 + 10).

Решите уравнение log 2 (2 – x) = log 2 (2 – 3x) +1.

Необходимо с правой стороны уравнения получить выражение вида:

log 2 (……)

Представляем 1 как логарифм с основанием 2:

1 = log 2 2

log с (ab) = log с a + log с b

log 2 (2 – x) = log 2 (2 – 3x) + log 2 2

Получаем:

log 2 (2 – x) = log 2 2 (2 – 3x)

Если log c a = log c b, то a = b, значит

2 – x = 4 – 6x

5x = 2

x = 0,4

Сделайте проверку.

Ответ: 0,4

Решите самостоятельно: Далее необходимо решить квадратное уравнение. Кстати,

корни равны 6 и – 4.

Корень «– 4″ не является решением, так как основание логарифма должно быть больше нуля, а при » – 4″ оно равно « – 5». Решением является корень 6. Сделайте проверку.

Ответ: 6.

Решите самостоятельно:

Решите уравнение log x –5 49 = 2. Если уравнение имеет более одного корня, в ответе укажите меньший из них.

Как вы убедились, никаких сложных преобразований с логарифмическими уравнениями нет. Достаточно знать свойства логарифма и уметь применять их. В задачах ЕГЭ, связанных с преобразованием логарифмических выражений, выполняются более серьёзные преобразования и требуются более глубокие навыки в решении. Такие примеры мы рассмотрим, не пропустите! Успехов вам!!!

С уважением, Александр Крутицких.

P.S: Буду благодарен Вам, если расскажете о сайте в социальных сетях.

Заключительные видео из длинной серии уроков про решение логарифмических уравнений. В этот раз мы будем работать в первую очередь с ОДЗ логарифма — именно из-за неправильного учета (или вообще игнорирования) области определения возникает большинство ошибок при решении подобных задач.

В этом коротком видеоуроке мы разберем применение формул сложения и вычитания логарифмов, а также разберемся с дробно-рациональными уравнениями, с которыми у многих учеников также возникают проблемы.

О чем пойдет речь? Главная формула, с которой я хотел бы разобраться, выглядит так:

log a (f g ) = log a f + log a g

Это стандартный переход от произведения к сумме логарифмов и обратно. Вы наверняка знаете эту формулу с самого начала изучения логарифмов. Однако тут есть одна заминка.

До тех пор, пока в виде переменных a , f и g выступают обычные числа, никаких проблем не возникает. Данная формула работает прекрасно.

Однако, как только вместоf и g появляются функции, возникает проблема расширения или сужения области определения в зависимости от того, в какую сторону преобразовывать. Судите сами: в логарифме, записанном слева, область определения следующая:

fg > 0

А вот в сумме, записанной справа, область определения уже несколько иная:

f > 0

g > 0

Данный набор требований является более жестким, чем исходный. В первом случае нас устроит вариант f 0 выполняется).

Итак, при переходе от левой конструкции к правой возникает сужение области определения. Если же сначала у нас была сумма, а мы переписываем ее в виде произведения, то происходит расширение области определения.

Другими словами, в первом случае мы могли потерять корни, а во втором — получить лишние. Это необходимо учитывать при решении реальных логарифмических уравнений.

Итак, первая задача:

[Подпись к рисунку]

Слева мы видим сумму логарифмов по одному и тому же основанию. Следовательно, эти логарифмы можно сложить:

[Подпись к рисунку]

Как видите, справа мы заменил ноль по формуле:

a = log b b a

Давайте еще немного преобразуем наше уравнение:

log 4 (x − 5) 2 = log 4 1

Перед нами каноническая форма логарифмического уравнения, мы можем зачеркнуть знак log и приравнять аргументы:

(x − 5) 2 = 1

|x − 5| = 1

Обратите внимание: откуда взялся модуль? Напомню, что корень из точного квадрата равен именно модулю:

[Подпись к рисунку]

Затем решаем классическое уравнение с модулем:

|f | = g (g > 0) ⇒f = ±g

x − 5 = ±1 ⇒x 1 = 5 − 1 = 4; x 2 = 5 + 1 = 6

Вот два кандидат на ответ. Являются ли они решением исходного логарифмического уравнения? Нет, ни в коем случае!

Оставить все просто так и записать ответ мы не имеем права. Взгляните на тот шаг, когда мы заменяем сумму логарифмов одним логарифмом от произведения аргументов. Проблема в том, что в исходных выражениях у нас стоят функции. Следовательно, следует потребовать:

х(х − 5) > 0; (х − 5)/х > 0.

Когда же мы преобразовали произведение, получив точный квадрат, требования изменились:

(x − 5) 2 > 0

Когда это требование выполняется? Да практически всегда! За исключением того случая, когда х − 5 = 0. Т.е. неравенство сведется к одной выколотой точке:

х − 5 ≠ 0 ⇒ х ≠ 5

Как видим, произошло расширение области определения, о чем мы и говорили в самом начале урока. Следовательно, могут возникнуть и лишние корни.

Как же не допустить возникновения этих лишних корней? Очень просто: смотрим на наши полученные корни и сравниваем их с областью определения исходного уравнения. Давайте посчитаем:

х (х − 5) > 0

Решать будем с помощью метода интервалов:

х (х − 5) = 0 ⇒ х = 0; х = 5

Отмечаем полученные числа на прямой. Все точки выколотые, потому что неравенство строгое. Берем любое число, больше 5 и подставляем:

[Подпись к рисунку]

На интересуют промежутки (−∞; 0) ∪ (5; ∞). Если мы отметим наши корни на отрезке, то увидим, что х = 4 нас не устраивает, потому что этот корень лежит за пределами области определения исходного логарифмического уравнения.

Возвращаемся к совокупности, вычеркиваем корень х = 4 и записываем ответ: х = 6. Это уже окончательный ответ к исходному логарифмическому уравнению. Все, задача решена.

Переходим ко второму логарифмическому уравнению:

[Подпись к рисунку]

Решаем его. Заметим, что первое слагаемое представляет собой дробь, а второе — ту же самую дробь, но перевернутую. Не пугайтесь выражения lgx — это просто десятичный логарифм, мы можем записать:

lgx = log 10 x

Поскольку перед нами две перевернутые дроби, предлагаю ввести новую переменную:

[Подпись к рисунку]

Следовательно, наше уравнение может быть переписано следующим образом:

t + 1/t = 2;

t + 1/t − 2 = 0;

(t 2 − 2t + 1)/t = 0;

(t − 1) 2 /t = 0.

Как видим, в числителе дроби стоит точный квадрат. Дробь равна нулю, когда ее числитель равен нулю, а знаменатель отличен от нуля:

(t − 1) 2 = 0; t ≠ 0

Решаем первое уравнение:

t − 1 = 0;

t = 1.

Это значение удовлетворяет второму требованию. Следовательно, можно утверждать, что мы полностью решили наше уравнение, но только относительно переменной t . А теперь вспоминаем, что такое t :

[Подпись к рисунку]

Получили пропорцию:

lgx = 2 lgx + 1

2 lgx − lgx = −1

lgx = −1

Приводим это уравнение к канонической форме:

lgx = lg 10 −1

x = 10 −1 = 0,1

В итоге мы получили единственный корень, который, по идее, является решением исходного уравнения. Однако давайте все-таки подстрахуемся и выпишем область определения исходного уравнения:

[Подпись к рисунку]

Следовательно, наш корень удовлетворяет всем требованиям. Мы нашли решение исходного логарифмического уравнения. Ответ: x = 0,1. Задача решена.

Ключевой момент в сегодняшнем уроке один: при использовании формулы перехода от произведения к сумме и обратно обязательно учитывайте, что область определения может сужаться либо расширяться в зависимости от того, в какую сторону выполняется переход.

Как понять, что происходит: сужение или расширение? Очень просто. Если раньше функции были вместе, а теперь стали по отдельности, то произошло сужение области определения (потому что требований стало больше). Если же сначала функции стояли отдельно, а теперь — вместе, то происходит расширение области определения (на произведение накладывается меньше требований, чем на отдельные множители).

С учетом данного замечания хотел бы отметить, что второе логарифмическое уравнение вообще не требует данных преобразований, т. е. мы нигде не складываем и не перемножаем аргументы. Однако здесь я хотел бы обратить ваше внимание на другой замечательный прием, который позволяет существенно упростить решение. Речь идет о замене переменной.

Однако помните, что никакие замены не освобождает нас от области определения. Именно поэтому после того были найдены все корни, мы не поленились и вернулись к исходному уравнению, чтобы найти его ОДЗ.

Часто при замене переменной возникает обидная ошибка, когда ученики находят значение t и думают, что на этом решение закончено. Нет, ни в коем случае!

Когда вы нашли значение t , необходимо вернуться к исходному уравнению и посмотреть, что именно мы обозначали этой буквой. В результате нам предстоит решить еще одно уравнение, которое, впрочем, будет значительно проще исходного.

Именно в этом состоит смысл введения новой переменной. Мы разбиваем исходное уравнение на два промежуточных, каждое из которых решается существенно проще.

Как решать «вложенные» логарифмические уравнения

Сегодня мы продолжаем изучать логарифмические уравнения и разберем конструкции, когда один логарифм стоит под знаком другого логарифма. Оба уравнения мы будем решать с помощью канонической формы.

Сегодня мы продолжаем изучать логарифмические уравнения и разберем конструкции, когда один логарифм стоит под знаком другого. Оба уравнения мы будем решать с помощью канонической формы. Напомню, если у нас есть простейшее логарифмическое уравнение вида log a f (x ) = b , то для решения такого уравнения мы выполняем следующие шаги. В первую очередь, нам нужно заменить число b :

b = log a a b

Заметьте: a b — это аргумент. Точно так же в исходном уравнении аргументом является функция f (x ). Затем мы переписываем уравнение и получаем вот такую конструкцию:

log a f (x ) = log a a b

Уже затем мы можем выполнить третий шаг — избавится от знака логарифма и просто записать:

f (x ) = a b

В результате мы получим новое уравнение. При этом никаких ограничений на функцию f (x ) не накладывается. Например, на ее месте также может стоять логарифмическая функция. И тогда мы вновь получим логарифмическое уравнение, которое снова сведем к простейшему и решим через каноническую форму.

Впрочем, хватит лирики. Давайте решим настоящую задачу. Итак, задача № 1:

log 2 (1 + 3 log 2 x ) = 2

Как видим, перед нами простейшее логарифмическое уравнение. В роли f (x ) выступает конструкция 1 + 3 log 2 x , а в роли числа b выступает число 2 (в роли a также выступает двойка). Давайте перепишем эту двойку следующим образом:

Важно понимать, что первые две двойки пришли к нам из основания логарифма, т. е. если бы в исходном уравнении стояла 5, то мы бы получили, что 2 = log 5 5 2 . В общем, основание зависит исключительно от логарифма, который изначально дан в задаче. И в нашем случае это число 2.

Итак, переписываем наше логарифмическое уравнение с учетом того, что двойка, которая стоит справа, на самом деле тоже является логарифмом. Получим:

log 2 (1 + 3 log 2 x ) = log 2 4

Переходим к последнему шагу нашей схемы — избавляемся от канонической формы. Можно сказать, просто зачеркиваем знаки log. Однако с точки зрения математики «зачеркнуть log» невозможно — правильнее сказать, что мы просто просто приравниваем аргументы:

1 + 3 log 2 x = 4

Отсюда легко находится 3 log 2 x :

3 log 2 x = 3

log 2 x = 1

Мы вновь получили простейшее логарифмическое уравнение, давайте снова приведем его к канонической форме. Для этого нам необходимо провести следующие изменения:

1 = log 2 2 1 = log 2 2

Почему в основании именно двойка? Потому что в нашем каноническом уравнении слева стоит логарифм именно по основанию 2. Переписываем задачу с учетом этого факта:

log 2 x = log 2 2

Снова избавляемся от знака логарифма, т. е. просто приравниваем аргументы. Мы вправе это сделать, потому что основания одинаковые, и больше никаких дополнительных действий ни справа, ни слева не выполнялось:

Вот и все! Задача решена. Мы нашли решение логарифмического уравнения.

Обратите внимание! Хотя переменная х и стоит в аргументе (т. е. возникают требования к области определения), мы никаких дополнительных требований предъявлять не будем.

Как я уже говорил выше, данная проверка является избыточной, если переменная встречается лишь в одном аргументе лишь одного логарифма. В нашем случае х действительно стоит лишь в аргументе и лишь под одним знаком log. Следовательно, никаких дополнительных проверок выполнять не требуется.

Тем не менее, если вы не доверяете данному методу, то легко можете убедиться, что х = 2 действительно является корнем. Достаточно подставить это число в исходное уравнение.

Давайте перейдем ко второму уравнению, оно чуть интересней:

log 2 (log 1/2 (2x − 1) + log 2 4) = 1

Если обозначить выражение внутри большого логарифма функцией f (x ), получим простейшее логарифмическое уравнение, с которого мы начинали сегодняшний видеоурок. Следовательно, можно применить каноническую форму, для чего придется представить единицу в виде log 2 2 1 = log 2 2.

Переписываем наше большое уравнение:

log 2 (log 1/2 (2x − 1) + log 2 4) = log 2 2

Изваляемся от знака логарифма, приравнивая аргументы. Мы вправе это сделать, потому что и слева, и справа основания одинаковые. Кроме того, заметим, что log 2 4 = 2:

log 1/2 (2x − 1) + 2 = 2

log 1/2 (2x − 1) = 0

Перед нами снова простейшее логарифмическое уравнение вида log a f (x ) = b . Переходим к канонической форме, т. е. представляем ноль в виде log 1/2 (1/2)0 = log 1/2 1.

Переписываем наше уравнение и избавляемся от знака log, приравнивая аргументы:

log 1/2 (2x − 1) = log 1/2 1

2x − 1 = 1

Опять же мы сразу получили ответ. Никаких дополнительных проверок не требуется, потому что в исходном уравнении лишь один логарифм содержит функцию в аргументе.

Следовательно, никаких дополнительных проверок выполнять не требуется. Мы можем смело утверждать, что х = 1 является единственным корнем данного уравнения.

А вот если бы во втором логарифме вместо четверки стояла бы какая-то функция от х (либо 2х стояло бы не в аргументе, а в основании) — вот тогда потребовалось бы проверять область определения. Иначе велик шанс нарваться на лишние корни.

Откуда возникают такие лишние корни? Этот момент нужно очень четко понимать. Взгляните на исходные уравнения: везде функция х стоит под знаком логарифма. Следовательно, поскольку мы записали log 2 x , то автоматически выставляем требование х > 0. Иначе данная запись просто не имеет смысла.

Однако по мере решения логарифмического уравнения мы избавляемся от всех знаков log и получаем простенькие конструкции. Здесь уже никаких ограничений не выставляется, потому что линейная функция определена при любом значении х.

Именно эта проблема, когда итоговая функция определена везде и всегда, а исходная — отнюдь не везде и не всегда, и является причиной, по которой в решении логарифмических уравнениях очень часто возникают лишние корни.

Но повторю еще раз: такое происходить лишь в ситуации, когда функция стоит либо в нескольких логарифмах, либо в основании одного из них. В тех задачах, которые мы рассматриваем сегодня, проблем с расширением области определения в принципе не существует.

Случаи разного основания

Этот урок посвящен уже более сложным конструкциям. Логарифмы в сегодняшних уравнениях уже не будут решаться «напролом» — сначала потребуется выполнить некоторые преобразования.

Начинаем решение логарифмических уравнений с совершенно разными основаниями, которые не являются точными степенями друг друга. Пусть вас не пугают подобные задачи — решаются они ничуть не сложнее, чем самые простые конструкции, которые мы разбирали выше.

Но прежде, чем переходить непосредственно к задачам, напомню о формуле решения простейших логарифмических уравнений с помощью канонической формы. Рассмотрим задачу вот такого вида:

log a f (x ) = b

Важно, что функция f (x ) является именно функцией, а в роли чисел а и b должны выступать именно числа (без всяких переменных x ). Разумеется, буквально через минуту мы рассмотрим и такие случаи, когда вместо переменных а и b стоят функции, но сейчас не об этом.

Как мы помним, число b нужно заменить логарифмом по тому же самому основанию а, которое стоит слева. Это делается очень просто:

b = log a a b

Разумеется, под словом «любое число b » и «любое число а» подразумеваются такие значения, которые удовлетворяют области определения. В частности, в данном уравнении речь идет лишь основание a > 0 и a ≠ 1.

Однако данное требование выполняется автоматически, потому что в исходной задаче уже присутствует логарифм по основанию а — оно заведомо будет больше 0 и не равно 1. Поэтому продолжаем решение логарифмического уравнения:

log a f (x ) = log a a b

Подобная запись называется канонической формой. Ее удобство состоит в том, что мы сразу можем избавиться от знака log, приравняв аргументы:

f (x ) = a b

Именно этот прием мы сейчас будем использовать для решения логарифмических уравнений с переменным основанием. Итак, поехали!

log 2 (x 2 + 4x + 11) = log 0,5 0,125

Что дальше? Кто-то сейчас скажет, что нужно вычислить правый логарифм, либо свести их к одному основанию, либо что-то еще. И действительно, сейчас нужно привести оба основания к одному виду — либо 2, либо 0,5. Но давайте раз и навсегда усвоим следующее правило:

Если в логарифмическом уравнении присутствуют десятичные дроби, обязательно переведите эти дроби из десятичной записи в обычную. Такое преобразование может существенно упростить решение.

Подобный переход нужно выполнять сразу, еще до выполнения каких-либо действий и преобразований. Давайте посмотрим:

log 2 (x 2 + 4x + 11) = log 1 /2 1/8

Что нам дает такая запись? Мы можем 1/2 и 1/8 представить как степень с отрицательным показателем:


[Подпись к рисунку]

Перед нами каноническая форма. Приравниваем аргументы и получаем классическое квадратное уравнение:

x 2 + 4x + 11 = 8

x 2 + 4x + 3 = 0

Перед нами приведенное квадратное уравнение, которое легко решается с помощью формул Виета. Подобные выкладки в старших классах вы должны видеть буквально устно:

(х + 3)(х + 1) = 0

x 1 = −3

x 2 = −1

Вот и все! Исходное логарифмическое уравнение решено. Мы получили два корня.

Напомню, что определять область определения в данном случае не требуется, поскольку функция с переменной х присутствует лишь в одном аргументе. Поэтому область определения выполняется автоматически.

Итак, первое уравнение решено. Переходим ко второму:

log 0,5 (5x 2 + 9x + 2) = log 3 1/9

log 1/2 (5x 2 + 9x + 2) = log 3 9 −1

А теперь заметим, что аргумент первого логарифма тоже можно записать в виде степени с отрицательным показателем: 1/2 = 2 −1 . Затем можно вынести степени с обеих сторон уравнения и разделить все на −1:

[Подпись к рисунку]

И вот сейчас мы выполнили очень важный шаг в решении логарифмического уравнения. Возможно, кто-то что-то не заметил, поэтому давайте я поясню.

Взгляните на наше уравнение: и слева, и справа стоит знак log, но слева стоит логарифм по основанию 2, а справа стоит логарифм по основанию 3. Тройка не является целой степенью двойки и, наоборот: нельзя записать, что 2 — это 3 в целой степени.

Следовательно, это логарифмы с разными основаниями, которые не сводятся друг к другу простым вынесением степеней. Единственный путь решения таких задач — избавиться от одного из этих логарифмов. В данном случае, поскольку мы пока рассматриваем довольно простые задачи, логарифм справа просто сосчитался, и мы получили простейшее уравнение — именно такое, о котором мы говорили в самом начале сегодняшнего урока.

Давайте представим число 2, которое стоит справа в виде log 2 2 2 = log 2 4. А затем избавимся от знака логарифма, после чего у нас остается просто квадратное уравнение:

log 2 (5x 2 + 9x + 2) = log 2 4

5x 2 + 9x + 2 = 4

5x 2 + 9x − 2 = 0

Перед нами обычное квадратное уравнение, однако оно не является приведенным, потому что коэффициент при x 2 отличен от единицы. Следовательно, решать мы его будем с помощью дискриминанта:

D = 81 − 4 5 (−2) = 81 + 40 = 121

x 1 = (−9 + 11)/10 = 2/10 = 1/5

x 2 = (−9 − 11)/10 = −2

Вот и все! Мы нашли оба корня, а значит, получили решение исходного логарифмического уравнения. Ведь в исходной задачи функция с переменной х присутствует лишь в одном аргументе. Следовательно, никаких дополнительных проверок на область определения не требуется — оба корня, которые мы нашли, заведомо отвечают всем возможным ограничениям.

На этом можно было бы закончить сегодняшний видеоурок, но в заключении я хотел бы сказать еще раз: обязательно переводите все десятичные дроби в обычные при решении логарифмических уравнений. В большинстве случаев это существенно упрощает их решение.

Редко, очень редко попадаются задачи, в которых избавление от десятичных дробей лишь усложняет выкладки. Однако в таких уравнениях, как правило, изначально видно, что избавляться от десятичных дробей не надо.

В большинстве остальных случаев (особенно если вы только начинаете тренироваться в решении логарифмических уравнений) смело избавляйтесь от десятичных дробей и переводите их в обычные. Потому что практика показывает, что таким образом вы значительно упростите последующее решение и выкладки.

Тонкости и хитрости решения

Сегодня мы переходим к более сложным задачам и будем решать логарифмическое уравнение, в основании которого стоит не число, а функция.

И пусть даже эта функция линейна — в схему решения придется внести небольшие изменения, смысл которых сводится к дополнительным требованиям, накладываемым на область определения логарифма.

Сложные задачи

Этот урок будет довольно длинным. В нем мы разберем два довольно серьезных логарифмических уравнения, при решении которых многие ученики допускают ошибки. За свою практику работы репетитором по математике я постоянно сталкивался с двумя видами ошибок:

  1. Возникновение лишних корней из-за расширения области определения логарифмов. Чтобы не допускать такие обидные ошибки, просто внимательно следите за каждым преобразованием;
  2. Потери корней из-за того, что ученик забыл рассмотреть некоторые «тонкие» случаи — именно на таких ситуациях мы сегодня и сосредоточимся.

Это последний урок, посвященный логарифмическим уравнениям. Он будет длинным, мы разберем сложные логарифмические уравнения. Устраивайтесь поудобней, заварите себе чай, и мы начинаем.

Первое уравнение выглядит вполне стандартно:

log x + 1 (x − 0,5) = log x − 0,5 (x + 1)

Сразу заметим, что оба логарифма являются перевернутыми копиями друг друга. Вспоминаем замечательную формулу:

log a b = 1/log b a

Однако у этой формулы есть ряд ограничений, которые возникают в том случае, если вместо чисел а и b стоят функции от переменной х:

b > 0

1 ≠ a > 0

Эти требования накладываются на основание логарифма. С другой стороны, в дроби от нас требуется 1 ≠ a > 0, поскольку не только переменная a стоит в аргументе логарифма (следовательно, a > 0), но и сам логарифм находится в знаменателе дроби. Но log b 1 = 0, а знаменатель должен быть отличным от нуля, поэтому a ≠ 1.

Итак, ограничения на переменную a сохраняется. Но что происходит с переменной b ? С одной стороны, из основания следует b > 0, с другой — переменная b ≠ 1, потому что основание логарифма должно быть отлично от 1. Итого из правой части формулы следует, что 1 ≠ b > 0.

Но вот беда: второе требование (b ≠ 1) отсутствует в первом неравенстве, посвященном левому логарифму. Другими словами, при выполнении данного преобразования мы должны отдельно проверить , что аргумент b отличен от единицы!

Вот давайте и проверим. Применим нашу формулу:

[Подпись к рисунку]

1 ≠ х − 0,5 > 0; 1 ≠ х + 1 > 0

Вот мы и получили, что уже из исходного логарифмического уравнения следует, что и а, и b должны быть больше 0 и не равны 1. Значит, мы спокойно можем переворачивать логарифмическое уравнение:

Предлагаю ввести новую переменную:

log x + 1 (x − 0,5) = t

В этом случае наша конструкция перепишется следующим образом:

(t 2 − 1)/t = 0

Заметим, что в числителе у нас стоит разность квадратов. Раскрываем разность квадратов по формуле сокращенного умножения:

(t − 1)(t + 1)/t = 0

Дробь равна нулю, когда ее числитель равен нулю, а знаменатель отличен от нуля. Но в числителе стоит произведение, поэтому приравниваем к нулю каждый множитель:

t 1 = 1;

t 2 = −1;

t ≠ 0.

Как видим, оба значения переменной t нас устраивают. Однако на этом решение не заканчивается, ведь нам требуется найти не t , а значение x . Возвращаемся к логарифму и получаем:

log x + 1 (x − 0,5) = 1;

log x + 1 (x − 0,5) = −1.

Давайте приведем каждое из этих уравнений к канонической форме:

log x + 1 (x − 0,5) = log x + 1 (x + 1) 1

log x + 1 (x − 0,5) = log x + 1 (x + 1) −1

Избавляемся от знака логарифма в первом случае и приравниваем аргументы:

х − 0,5 = х + 1;

х − х = 1 + 0,5;

Такое уравнение не имеет корней, следовательно, первое логарифмическое уравнение также не имеет корней. А вот со вторым уравнением все намного интересней:

(х − 0,5)/1 = 1/(х + 1)

Решаем пропорцию — получим:

(х − 0,5)(х + 1) = 1

Напоминаю, что при решении логарифмических уравнений гораздо удобней приводить все десятичные дроби обычные, поэтому давайте перепишем наше уравнение следующим образом:

(х − 1/2)(х + 1) = 1;

x 2 + x − 1/2x − 1/2 − 1 = 0;

x 2 + 1/2x − 3/2 = 0.

Перед нами приведенное квадратное уравнение, оно легко решается по формулам Виета:

(х + 3/2) (х − 1) = 0;

x 1 = −1,5;

x 2 = 1.

Получили два корня — они являются кандидатами на решение исходного логарифмического уравнения. Для того чтобы понять, какие корни действительно пойдут в ответ, давайте вернемся к исходной задаче. Сейчас мы проверим каждый из наших корней на предмет соответствия области определения:

1,5 ≠ х > 0,5; 0 ≠ х > −1.

Эти требования равносильны двойному неравенству:

1 ≠ х > 0,5

Отсюда сразу видим, что корень х = −1,5 нас не устраивает, а вот х = 1 вполне устраивает. Поэтому х = 1 — окончательное решение логарифмического уравнения.

Переходим ко второй задаче:

log x 25 + log 125 x 5 = log 25 x 625

На первый взгляд может показаться, что у всех логарифмов разные основания и разные аргументы. Что делать с такими конструкциями? В первую очередь заметим, что числа 25, 5 и 625 — это степени 5:

25 = 5 2 ; 625 = 5 4

А теперь воспользуемся замечательным свойством логарифма. Дело в том, что можно выносить степени из аргумента в виде множителей:

log a b n = n ∙ log a b

На данное преобразование также накладываются ограничения в том случае, когда на месте b стоит функция. Но у нас b — это просто число, и никаких дополнительных ограничений не возникает. Перепишем наше уравнение:

2 ∙ log x 5 + log 125 x 5 = 4 ∙ log 25 x 5

Получили уравнение с тремя слагаемыми, содержащими знак log. Причем аргументы всех трех логарифмов равны.

Самое время перевернуть логарифмы, чтобы привести их к одному основанию — 5. Поскольку в роли переменной b выступает константа, никаких изменений области определения не возникает. Просто переписываем:


[Подпись к рисунку]

Как и предполагалось, в знаменателе «вылезли» одни и те же логарифмы. Предлагаю выполнить замену переменной:

log 5 x = t

В этом случае наше уравнение будет переписано следующим образом:

Выпишем числитель и раскроем скобки:

2 (t + 3) (t + 2) + t (t + 2) − 4t (t + 3) = 2 (t 2 + 5t + 6) + t 2 + 2t − 4t 2 − 12t = 2t 2 + 10t + 12 + t 2 + 2t − 4t 2 − 12t = −t 2 + 12

Возвращаемся к нашей дроби. Числитель должен быть равен нулю:

[Подпись к рисунку]

А знаменатель — отличен от нуля:

t ≠ 0; t ≠ −3; t ≠ −2

Последние требования выполняются автоматически, поскольку все они «завязаны» на целые числа, а все ответы — иррациональные.

Итак, дробно-рациональное уравнение решено, значения переменной t найдены. Возвращаемся к решению логарифмического уравнения и вспоминаем, что такое t :

[Подпись к рисунку]

Приводим это уравнение к канонической форме, получим число с иррациональной степенью. Пусть это вас не смущает — даже такие аргументы можно приравнять:

[Подпись к рисунку]

У нас получилось два корня. Точнее, два кандидата в ответы — проверим их на соответствие области определения. Поскольку в основании логарифма стоит переменная х, потребуем следующее:

1 ≠ х > 0;

С тем же успехом утверждаем, что х ≠ 1/125, иначе основание второго логарифма обратится в единицу. Наконец, х ≠ 1/25 для третьего логарифма.

Итого мы получили четыре ограничения:

1 ≠ х > 0; х ≠ 1/125; х ≠ 1/25

А теперь вопрос: удовлетворяют ли наши корни указанным требованиям? Конечно удовлетворяют! Потому что 5 в любой степени будет больше нуля, и требование х > 0 выполняется автоматически.

С другой стороны, 1 = 5 0 , 1/25 = 5 −2 , 1/125 = 5 −3 , а это значит, что данные ограничения для наших корней (у которых, напомню, в показателе стоит иррациональное число) также выполнены, и оба ответа являются решениями задачи.

Итак, мы получили окончательный ответ. Ключевых моментов в данной задаче два:

  1. Будьте внимательны при перевороте логарифма, когда аргумент и основание меняются местами. Подобные преобразования накладывают лишние ограничения на область определения.
  2. Не бойтесь преобразовывать логарифмы: их можно не только переворачивать, но и раскрывать по формуле суммы и вообще менять по любым формулам, которые вы изучали при решении логарифмических выражений. Однако при этом всегда помните: некоторые преобразования расширяют область определения, а некоторые — сужают.

Логарифмические уравнения — основы | О математике понятно

        Логарифмические уравнения — штука, вообще говоря, не самая простая. Слишком уж много их. Простых, суперсложных, всяких.) Кроме того, для их безошибочного решения необходимы пристальное внимание и приличный запас знаний по другим, смежным темам.

        А именно:

        1. Логарифмы

        Что такое логарифм и что с ним можно делать?

        Ограничения в логарифмах. Свойства логарифмов.

 

        2. Степени

        — Базовые действия со степенями.

        — Работа с отрицательными и дробными показателями.

 

        3. Уравнения

        — Тождественные преобразования уравнений.

        — Что такое ОДЗ и куда её пристраивать?

       

        4. Алгебра и её общие правила и законы

        А именно: приведение подобных, разложение на множители, формулы сокращённого умножения и т.д.

        Именно эти четыре ножки позволяют сидеть твёрдо, надёжно и не падать на ровном месте. Ибо больно это.) Разберёмся?

 

Что такое логарифмическое уравнение? Примеры.

        Как и намекает название, логарифмическое уравнение — это уравнение с логарифмами. Но не просто с логарифмами, а такое, где иксы находятся исключительно внутри логарифмов. И только там! Это крайне важно.

        Иксы (чаще всего) встречаются в аргументе логарифма:

        log2x = log23

        log5(2x-1) = log5x

        log4(x+1) = 3

        lg(x2–5x+16) = 1

        ln(x+3)+ln(x-5) = ln(3x-1)

 

        Иксы иногда могут быть и в основании логарифма:

        log2x+527 = 3

 

        Или даже и там и там одновременно:

        logx-1(x+5) = 2

 

        Где угодно могут быть иксы. Лишь бы внутри логарифмов. Если, вдруг, в уравнении икс вылезет где-то снаружи, что-нибудь типа:

        log3x = 11-x ,

        то такое уравнение будет уже уравнением смешанного типа. Чётких правил решения такие уравнения не имеют, поэтому в этом уроке мы их рассматривать и не будем. Радуйтесь.)

        Кстати, может попасться и такое уравнение, где внутри логарифмов сидят только числа. Например, такое:

        3x+2 = lg400 — lg4

        Что тут скажешь? Халява, если попалось такое! Ибо логарифм с числами — это просто какое-то число. Всего-навсего. Для расправы с таким монстром достаточно лишь знать свойства логарифмов. И всё. Каких-то специальных приёмов и правил, предназначенных именно для решения логарифмических уравнений, здесь не нужно.

        Кстати, не следует думать, что внутри логарифмов могут стоять лишь линейные да квадратные выражения с иксами. Любые могут стоять. И дробные, и показательные, и тригонометрические — какие угодно, насколько позволяет фантазия составителей примера! Просто подавляющее большинство школьных заданий (процентов 90) — именно на линейные и квадратные выражения внутри логарифмов. Да и писать мне их проще, чем всякие там дроби, корни, синусы и прочую экзотику. Да и начинать знакомство обычно принято с самого простого.)

        Итак, что такое логарифмическое уравнение — уяснили. Как же их решать?

 

Как решать логарифмические уравнения? Простейшие примеры.

        Как уже говорилось выше, единого рецепта решения логарифмических уравнений на все случаи жизни в математике нет. Много их, уравнений. Самых разных…

        Но волноваться не стоит. Ибо среди всего богатого многообразия логарифмических уравнений выделяются такие, решать которые очень и очень легко. В уме, фактически. Такие уравнения так и называются — простейшими. Именно с них и начнём. Для их решения желательно лишь иметь общее представление о логарифме вообще, не более того. Зачем? Гм… Согласитесь, как-то неловко браться за решение логарифмического уравнения, даже понятия не имея что такое логарифм… Самонадеянно, я бы сказал. )

        Так что же это за уравнения?

        Это уравнения типа:

        log2x = log23

        log5(2x-1) = log5x

        log4(x+1) = 3

        log2x+527 = 3

        И так далее и тому подобное…

        А теперь собираем волю в кулак и вникаем в простые вещи. В чём суть? Процесс любого (да-да, именно любого!) логарифмического уравнения заключается в переходе от уравнения с логарифмами к уравнению без них. В простейших уравнениях этот переход осуществляется всего за один шаг. На то они и простейшие.) Как именно? Это только на конкретных примерах показать можно.

        Решаем первое уравнение по списку:

        log2x = log23

        Для решения этого крутого примера знать почти ничего и не надо, да. Обычная интуиция.) Что нам сильнее всего не нравится в этом примере? Что-что… Логарифмы не нравятся! Верно. Избавиться бы от них… Но как? Смотрим на пример. Смотрим и… прям таки хочется взять, да и выкинуть логарифмы вообще! А можно? Новость хорошая: да, можно! Математика не возражает.)

        Убираем логарифмы и сразу получаем ответ:

        х = 3

        Здорово, правда? И, главное, просто и быстро! И так нужно поступать всегда. Выкидывание логарифмов подобным образом — ключевая идея решения любого логарифмического уравнения. Простого, сложного — неважно. В математике эта приятная операция (выкидывание логарифмов) носит своё специальное название — потенцирование. Но на такую ликвидацию есть набор жёстких правил. Перечислю их.

        Убирать логарифмы (т.е. потенцировать) можно, если:

        1. У них одинаковые основания;

        2. Логарифмы слева и справа чистые и стоят в гордом одиночестве.

 

        Разберёмся с этими двумя пунктиками поподробнее. Иначе потом эти пунктики будут жестоко мстить на контрольных и экзаменах.

        Например, уравнение:

        log2x = log0,52

        Можно здесь убирать логарифмы? Нет! Основания разные: слева стоит логарифм по основанию 2, а справа — по основанию 0,5. Разные числа. А нужны — одинаковые. Не катит…

        Или такой пример:

        lgx + lg(x+1) = lg(x2+4)

        Здесь тоже нельзя потенцировать уравнение. Пусть даже все три логарифма по одному основанию (десятке), но слева не одинокий логарифм, их там два.

        Или такое:

        log3(2x-1) = 2log3x

        А вот здесь нельзя потенцировать по другой причине: двойка справа не позволяет. Коэффициент, понимаешь. Мелочь, а тем не менее существенная.) Правило — штука жёсткая.

        Одним словом, убирать логарифмы можно тогда и только тогда, когда логарифмическое уравнение выглядит именно (и только!) вот так:

        

        Основания «а» — одинаковые. Причём, даже не обязательно числовые. Могут быть с иксом, могут быть без икса — не суть. Важно, что одинаковые. А вот в скобках, на месте многоточий, могут стоять совершенно любые выражения с иксом — простые, сложные — какие угодно. Это неважно. Важно другое. А именно — то, что после ликвидации логарифмов у нас остаётся более простое уравнение без логарифмов. Конечно же, предполагается, что решать линейные, квадратные, дробные, показательные, иррациональные, тригонометрические и прочие уравнения без логарифмов вы уже умеете.

        Теперь, вооружившись ценными познаниями, решаем следующий пример по списку:

        log5(2x-1) = log5x

        Все условия для потенцирования выполняются: основания одинаковые, логарифмы слева и справа чистые. Потенцируем и решаем простенькое линейное уравнение:

        2x-1 = x

        x = 1

        Вот и всё.) Как видите, вся логарифмическая часть решения уравнения заключается только в ликвидации логарифмов. И всё! А дальше работаем с более простым линейным уравнением, уже без логарифмов. Элементарно.

        Остальные уравнения из нашего списка уже так просто не решить. Здесь уже надо знать, что такое логарифм.

        Например, типичное задание из ЕГЭ — это третье уравнение:

        log4(x+1) = 3

        Слева стоит логарифм от чего-то там, а справа — число. Вспоминаем элементарный смысл логарифма. А именно — то, что этот самый логарифм — это какое-то число, в которое надо возвести основание (т.е. четыре), чтобы получить аргумент логарифма (т.е. х+1).

        Но это число нам известно! Оно равно трём! Прямо по нашему уравнению.)

        Стало быть, имеем полное право записать:

        43 = х+1

        Всё. Логарифм исчез и осталось безобидное линейное уравнение:

        64 = х+1

        х = 63

        Это уравнение мы решили, пользуясь только определением логарифма. Что, потенцированием решать проще? Да, согласен! Если вы умеете делать из числа логарифм по любому основанию, то никаких проблем. Если не умеете, читайте третий урок про логарифмы. Там всё популярно изложено.

        Итак, перед нами то же самое уравнение:

        log4(x+1) = 3

        Для потенцирования нам необходимо добиться, чтобы в уравнении слева и справа стояли логарифмы по одному и тому же основанию. Для этого справа из тройки сделаем логарифм по основанию четыре. Как именно? По нашей старой доброй технологии, подробно описанной по ссылке выше.

        Вот так:

        3 = log443

        Тогда получим следующее:

        log4(x+1) = log443

        Вот мы и получили то что хотели. Слева логарифм по основанию четыре, справа тоже. С чувством глубокого удовлетворения потенцируем и приходим к тому же самому ответу:

        х+1 = 43

        х+1 = 64

        х = 63

        Как видите, и так и сяк решать можно. Как хотите, так и решайте. Кому-то хватает простого определения логарифма, а кому-то удобно потенцирование. Как говорится, о вкусах не спорят. Ещё раз напоминаю, что умение делать из числа логарифм — весьма и весьма полезная штука в логарифмических уравнениях. И особенно — в неравенствах! Не пренебрегаем.)

        И, наконец, последнее уравнение:

        log2x+527 = 3

        Здесь отличие от предыдущих примеров заключается в том, что икс стоит не в аргументе, а в основании логарифма. Ну и что? Решается уравнение совершенно аналогично, по определению логарифма:

        (2х+5)3 = 27

        2х+5 = 3

        2х = -2

        х = -1

        И все дела.)

        Кстати, попробуйте решить этот же пример через потенцирование. Удивитесь, но получите то же самое.)

 

        Итак, с простейшими уравнениями всё предельно ясно. Работаем либо напрямую через потенцирование (если логарифмы слева и справа), либо по определению логарифма (если слева логарифм, а справа число).

        А теперь рассмотрим несколько уравнений чуть посложнее. Таких, которые напрямую через ликвидацию логарифмов не решаются. Их сначала надо немного подготовить к ликвидации. Как готовить? Свойства логарифмов — вот он, ключевой рецепт!

 

        Пользуемся свойствами логарифмов!

        Проскакивала у нас выше парочка уравнений.

        Например:

        lgx + lg(x+1) = lg(x2+4)

        Что делать? Убирать логарифмы пока нельзя, против правил будет: слева сумма логарифмов. А нужен — одинокий. Как выкрутиться? Да свойства логарифмов вспомнить! Мы же знаем, что сумму двух логарифмов всегда можно превратить в один логарифм, но от произведения. Если основания одинаковые, конечно.

        Вот и превращаем по формуле:

        

        Вот и всё. Заменяем сумму логарифмов на логарифм произведения:

        lg(x2+x) = lg(x2+4)

        Что, десятичные логарифмы (lg) вас смущают? Зря! Ещё раз напоминаю, что базовые логарифмические формулы, операции и свойства распространяются на все виды логарифмов без исключения. Десятичные, натуральные, по основанию 13, по основанию «пи» — любые! Так что можно складывать логарифмы по формуле и потенцировать совершенно безболезненно. Лишь бы основание в процессе применения формул и перед потенцированием оставалось одинаковым. В нашем случае — десятка.

        Смело потенцируем, решаем:

        x2+x = x2+4

        х = 4

        Всё!

 

        Запоминаем:

        Десятичные (lg) и натуральные (ln) логарифмы ни по определению, ни по свойствам ничем не отличаются от обычных!

 

        Ещё один примерчик:

        log3(2x-1) = 2log3x

        Опять же, потенцировать уравнение нельзя: справа нужен чистый логарифм, а двойка мешает. И снова нам на помощь приходят свойства логарифмов! Не нравится двойка? Подумаешь, проблема! А мы её внутрь логарифма спрячем.)

        Вот так:

        2log3x = log3x2

        И все дела, логарифм от коэффициента «очистили». ) А дальше — по накатанной колее, подставляем, потенцируем:

        log3(2x-1) = log3x2

        2х — 1 = х2

        x2 — 2x + 1 = 0

        Решаем простецкое квадратное уравнение и получаем единственный корень

        х = 1

        Как видите, тоже ничего сложного. Одно-два простых свойства логарифмов и — готово дело.) Но… бывают и сюрпризы, да. Например, некоторые ученики успешно решают уравнения, где только логарифмы, но зависают на уравнениях с постоянными числами.

 

        Делаем логарифмы из постоянных чисел!

        Например такое уравнение:

        log2(3x-2) + 3 = log2(14x+1)

        Куда пристегнуть тройку? Никуда не надо пристёгивать, надо снова из тройки логарифм сделать! По нашей излюбленной схеме:

        3 = log223 = log28

        А вот теперь логарифмические формулы в уравнении отлично идут!

        log2(3x-2) + log28 = log2(14x+1)

        log2(8(3x-2)) = log2(14x+1)

        Убираем логарифмы и дорешиваем элементарщину:

        8(3x-2) = 14x+1

        24x-16 = 14x+1

        10x = 17

        x = 1,7

        И всего делов-то. А вот без превращения тройки в логарифм это уравнение так просто не решилось бы!

 

        Запоминаем:

        Любое число (или даже выражение!) можно превратить в логарифм по любому основанию.

 

        Разберём ещё один сюрприз. Это когда уравнение с виду простенькое, но основания логарифмов — разные.

 

        Разные основания — что делать?

        Например:

        log9x + 2log3x = 5

        Что, тупик? Складывать логарифмы по формуле нельзя, а уж убирать совсем — тем более. Основания не разрешают. Они… разные, да.) Тройка и девятка.

        Вот и делаем их одинаковыми!

        Классический метод — действия со степенями. В нашем случае проблема решается очень просто. Ибо из девятки тройку сделать (и наоборот) — пара пустяков. Ведь 9=32, не так ли? Про родню по степени помните?

        А если вспомнить формулку, избавляющую нас от степени в основании

        

        то совсем отлично получится:

        

        

        Воот. Основания выровняли. Правда, коэффициенты вылезли — ну и что? Можно, конечно, их внутрь логарифмов (в аргументы) степенями загнать и далее по формуле сложения свести всё к одному логарифму, но, может, есть способ попроще? Как вы думаете?

        Да! Просто привести подобные, как с буквами! Логарифмы-то одинаковые.) А алгебра — штука универсальная, и ей неважно, логарифмы под буквами скрываются или синусы, яблоки, зарплата, время. Для неё это всегда будет 0,5а+2а = 2,5а.

        Вот и пишем:

        2,5log3x = 5

        Остались пустяки — поделить на 2,5 (базовые преобразования никто не отменял) и решить простейшее уравнение:

        log3x = 2

        x = 9

        Здесь нас, конечно же, спасла родня по тройке. А если основания не родственные? Скажем, не 3 и 9, а 3 и 7? Тогда, скорее всего, преобразования более хитрые. Но это — тема отдельного урока.)

 

        Не забываем действия со степенями!

        Вроде бы, столько долбим эти несчастные степени, а ступор у некоторых иногда случается. Как только приходится сталкиваться с отрицательными да дробными показателями…

        Например, решали мы такое уравнение:

        log4(x+1) = 3

        Сейчас я поменяю в нём всего одно число. Вот такое уравнение теперь решим:

        log4(x+1) = -0,5

        И что теперь делать? С тройкой-то всё просто решалось, а тут -0,5. Как ни странно, делаем всё то же самое!

        Прямо по смыслу логарифма пишем:

        х+1 = 4-0,5

        А вот тут многие и зависнут. Что такое степень 0,5? Да ещё и с минусом…

        Что-что… Со степенями у вас проблемы — вот что! Коли такие вопросы волнуют в старших классах. Срочно повторить отрицательные и дробные показатели!

        А кто со степенями в мире и согласии, тот лишь скупо улыбнётся и твёрдой рукой запишет:

        

        Дальше всё ясно:

        х+1 = 0,5

        х = -0,5

 

        Что мы видим? Мы видим, что, каким бы ни было наше уравнение — простым, посложнее, суперсложным (о них в других уроках), мы всегда добиваемся одной и той же цели. А именно — чтобы слева и справа в уравнении стояли чистые логарифмы по одному и тому же основанию. Или ситуации «слева — чистый логарифм, справа — число». То есть, простейшего уравнения. А дальше — дело техники.

        Итак, самое главное по простейшим уравнениям и уравнениям, сводящимся к оным.

        Запоминаем:

        1) Основной принцип решения простейших логарифмических уравнений — это потенцирование левой и правой частей (мысленная ликвидация логарифмов). Или решение через определение логарифма.

        2) Простейшие логарифмические уравнения — это финишная прямая при решении любых других более сложных уравнений с логарифмами.

        3) Любое сложное логарифмическое уравнение надо пробовать свести к простейшему (или к их комбинации). Либо базовыми тождественными преобразованиями уравнений, либо с помощью свойств логарифмов и действий со степенями.

 

        А теперь, как всегда, набиваем руку на самых простых примерах.

        Найдите корень или сумму корней (если их несколько) уравнений:

        ln(3x+7) = ln(2x-1)

        log2x = 5

        log7(x2+28) = log7(11x)

        lg(x2–5x+16) = 1

        log0,1(3x+7) = -2

        logx2 = 0,25

        ln(e3+3x-6) = 3

 

        Ответы (в беспорядке):

        16; 32; 5; 2; 31; 8; 11

 

        Получилось? Великолепно! Куда уж проще-то… Что? Число «е» смутило в последнем уравнении? А вы вспомните, что служит основанием натурального логарифма (ln) — и будет вам счастье.)

 

        Примерчики посложнее (но ненамного). Здесь уже подключаем свойства логарифмов вкупе с тождественными преобразованиями уравнений.

        Решить уравнения и найти сумму корней (если их несколько):

        lg(x2+x+1) = 2lg(x+1)

        log5(7x+4) — log5(2x-1) = 1

        log4x + 3log2x = 7

        log2(2x2–x+2) = log2x+2

 

        Ответы (в беспорядке):

        2,5; 3; 0; 4

 

        И это вышло? Блеск! Значит, первую часть в решении логарифмических уравнений (хотя бы самых простых) вы уже освоили хорошо. Поздравляю! Осталось теперь разобраться со второй частью — и мы с вами будем уже во всеоружии. Можно будет и более хитрые примеры решать.)

        Как видите, всё просто. Сводим уравнение к простейшему за один-два шага, выкидываем логарифмы, решаем себе и записываем ответ. Это хорошая новость.

        А теперь — плохая новость. Успех в решении логарифмических уравнений этого урока вовсе не гарантирует вам успех в решении всех остальных уравнений. Даже простеньких на вид, подобных этим. Увы и ах…

        Так в чём же дело? Это и есть та самая вторая часть решения логарифмических уравнений (любых!). И более того, я бы даже назвал это основной проблемой в решении логарифмических уравнений. Да и неравенств тоже.

        Разберёмся с этой проблемой основательно! И решим её. Обязательно! В следующем уроке. Здесь ничего говорить не буду, дабы не рушить идиллию и не пугать народ.)

 

 

Логарифмические уравнения

Логарифмические уравнения

Определение. Логарифмические уравнения — это уравнения, содержащие переменную под знаком логарифма.

Например: log3 (3x — 2) = 4.

Решение логарифмических уравнений основывается на определении логарифма, свойствах логарифмической функции и свойствах логарифма

Основные методы решения логарифмических уравнений

  1. log a f(x) = b            f(x) = ab     a > 0, a ≠ 1.

  2. log f(x) g(x) = b   <=>   (f(x))b = g(x)f(x) > 0f(x) ≠ 1g(x) > 0

  3. log a f(x) = log a g(x)   <=>   f(x) = g(x)f(x) > 0   или   f(x) = g(x)g(x) > 0

  4. log f(x) g(x) = logf(x) h(x)   <=>   g(x) = h(x)f(x) > 0f(x) ≠ 1g(x) > 0   или   g(x) = h(x)f(x) > 0f(x) ≠ 1h(x) > 0

  5. loga f(x) + loga g(x) = loga h(x)   <=>   log a (f(x) g(x)) = log a h(x)f(x) > 0g(x) > 0h(x) > 0

  6. loga f(x) — loga g(x) = loga h(x)   <=>   log a f(x)g(x) = log a h(x)f(x) > 0g(x) > 0h(x) > 0

  7. n loga f(x) = loga h(x)   <=>   log a f(x)n = log a h(x)f(x) > 0h(x) > 0

  8. log a (f(x) g(x)) = log a |f(x)| + log a |g(x)|

    log af(x)g(x) = log a |f(x)| — log a |g(x)|

    log a (f(x))2n = 2n log a |f(x)|

Примеры решения логарифмических уравнений

1.

Использование определения логарифма

Пример 1.

Решить уравнение log27 x = 23.

Сначала найдем область допустимых значений уравнения (ОДЗ): x > 0

Преобразуем логарифмическое уравнение и выполним вычисления:

log27 x = 23   <=>   272/3 = x

x = 272/3 = (33)2/3 = 32 = 9

Ответ: x = 9.

Пример 2.

Решить уравнение log2 (x — 3) = 4.

Найдем ОДЗ уравнения: x — 3 > 0   =>   x > 3

Из определения логарифма получим:

x — 3 = 24 = 16

x = 16 + 3 = 19

Ответ: x = 19.

Пример 3.

Решить уравнение logx (2x2 — 3x — 4) = 2.

Найдем ОДЗ уравнения: x > 0x ≠ 12x2 — 3x — 4 > 0

Из определения логарифма получим:

x2 = 2x2 — 3x — 4   =>   x2 — 3x — 4 = 0

Используя теорему Виета легко найти корни уравнения x2 — 3x — 4 = 0

x1 = 4, x2 = -1

Выберем корни входящие в ОДЗ:

x1 = 4 — удовлетворяет всем условиям ОДЗ:

4 > 04 ≠ 12·42 — 3·4 — 4 = 32 — 12 — 4 = 16 > 0

x2 = -1 — не удовлетворяет первому условию из ОДЗ:

-1 < 0-1 ≠ 12·(-1)2 — 3·(-1) — 4 = 2 + 3 — 4 = 1 > 0

Ответ: x = 4.


2. Метод потенцирования

Пример 4.

Решить уравнение log3 (x2 — 4x — 5) = log3 (7 — 3x).

Найдем ОДЗ уравнения: x2 — 4x — 5 > 07 — 3x > 0

Заменим логарифмическое уравнение равносильным:

x2 — 4x — 5 = 7 — 3x

x2 — x — 12 = 0

Используя теорему Виета легко найти корни уравнения x2 — x — 12 = 0

x1 = 4, x2 = -3

Выберем корни входящие в ОДЗ:

x1 = 4 — не удовлетворяет условиям ОДЗ:

42 — 4·4 — 5 = -5 < 07 — 3·4 = -5 < 0

x2 = -3 — удовлетворяет условиям ОДЗ:

(-3)2 — 4·(-3) — 5 = 16 > 07 — 3·(-3) = 16 > 0

Ответ: x = -3.

Пример 5.

Решить уравнение lg (x — 9) + lg (2x — 1) = 2.

Найдем ОДЗ уравнения: x — 9 > 02x — 1 > 0   =>   x > 9x > 0. 5   =>   x ϵ (9; +∞)

Так как lg 100 = 2, то

lg (x — 9) + lg (2x — 1) = lg 100

Используем свойство, что сумма логарифмов равна логарифму произведения

lg (x — 9)(2x — 1) = lg 100

Заменим логарифмическое уравнение равносильным:

(x — 9)(2x — 1) = 100

2x2 — 19x + 9 = 100

2x2 — 19x — 91 = 0

Решим квадратное уравнение:

D = 192 — 4·2·(-91) = 1089

x1 = 19 + √10892·2 = 19 + 334 = 13

x2 = 19 — √10892·2 = 19 — 334 = -3.5

ОДЗ удовлетворяет только один корень x = 13

Ответ: x = 13.


3. Метод замены переменной, сведение логарифмического уравнения к алгебраическому

Пример 6.

Решить уравнение 112lg2 x = 13 — 14lg x.

ОДЗ: x > 0.

Выполним замену переменной lg x = t:

112t2 = 13 — 14t

t2 = 4 — 3t

t2 + 3t — 4 = 0

Используя теорему Виета легко найти корни уравнения

t1 = -4

t2 = 1

Вернемся к переменной x

lg x = -4lg x = 1   =>   x = 10-4x = 10

Ответ: уравнение имеет два корня x1 = 0. 0001 и x2 = 10.


4. Сведение логарифмического уравнения к одной основе

Пример 7.

Решить уравнение log4 x + log1/16 x + log8 x3 = 5.

ОДЗ: x > 0.

Используя свойства логарифмов сведем логарифмы в уравнении к основе 2:

12log2 x — 14log2 x + log2 x = 5

(12 — 14 + 1)log2 x = 5

54log2 x = 5

log2 x = 4

x = 24 = 16

Ответ: x = 16.


5. Логарифмирование обоих частей уравнения

Пример 8.

Решить уравнение xlg x = 100x.

ОДЗ: x > 0.

Прологарифмируем обе части уравнения по основе 10, и используем свойства логарифма степени и частного:

lg xlg x = lg 100x

lg x · lg x = lg 100 — lg x

lg2 x + lg x — 2 = 0

Выполним замену переменной lg x = t:

t2 + x — 2 = 0

Используя теорему Виета легко найти корни уравнения

t1 = -2

t2 = 1

Вернемся к переменной x

lg x = -2lg x = 1   =>   x = 10-2x = 10

Ответ: уравнение имеет два корня x1 = 0. 01 и x2 = 10.


6. Использование монотонности при решении логарифмических уравнений

Пример 9.

Решить уравнение log5 (x + 3) = 3 — x.

ОДЗ: x > 0.

y = log5 (x + 3) — монотонно возрастающая функция;

y = 3 — x — монотонно убывающая функция;

Так как первая функция монотонно возрастающая, а вторая монотонно убывающая, то они имеют одну точку пересечения, которая будет решением исходного уравнения.

Подбором найдем решение:

При x = 2   =>   log5 (2 + 3) = 1; 3 — 2 = 1

Ответ: x = 2.

Логарифмы Логарифм числа, основное логарифмическое тождество Формулы и свойства логарифмов Логарифм произведения. Сумма логарифмов Логарифм частного. Разность логарифмов Логарифм степени Логарифм корня Логарифмирование Потенцирование Десятичный логарифм Натуральный логарифм Число е Логарифмическая функция Логарифмические уравнения Логарифмические неравенства

Корень уравнения с логарифмами примеры.

Логарифмы: примеры и решения

Логарифмические уравнения. Продолжаем рассматривать задачи из части В ЕГЭ по математике. Мы с вами уже рассмотрели решения некоторых уравнений в статьях « » , « » . В этой статье рассмотрим логарифмические уравнения. Сразу скажу, что никаких сложных преобразований при решении таких уравнений на ЕГЭ не будет. Они просты.

Достаточно знать и понимать основное логарифмическое тождество, знать свойства логарифма. Обратите внимание на то, то после решения ОБЯЗАТЕЛЬНО нужно сделать проверку — подставить полученное значение в исходное уравнение и вычислить, в итоге должно получиться верное равенство.

Определение :

Логарифмом числа a по основанию b называется показатель степени, в который нужно возвести b, чтобы получить a.


Например:

Log 3 9 = 2, так как 3 2 = 9

Свойства логарифмов:

Частные случаи логарифмов:

Решим задачи. В первом примере мы сделаем проверку. В последующих проверку сделайте самостоятельно.

Найдите корень уравнения: log 3 (4–x) = 4

Так как log b a = x b x = a, то

3 4 = 4 – x

x = 4 – 81

x = – 77

Проверка:

log 3 (4–(–77)) = 4

log 3 81 = 4

3 4 = 81 Верно.

Ответ: – 77

Решите самостоятельно:

Найдите корень уравнения: log 2 (4 – x) = 7

Найдите корень уравнения log 5 (4 + x) = 2

Используем основное логарифмическое тождество.

Так как log a b = x b x = a, то

5 2 = 4 + x

x =5 2 – 4

x = 21

Проверка:

log 5 (4 + 21) = 2

log 5 25 = 2

5 2 = 25 Верно.

Ответ: 21

Найдите корень уравнения log 3 (14 – x) = log 3 5.

Имеет место следующее свойство, смысл его таков: если в левой и правой частях уравнения имеем логарифмы с одинаковым основанием, то можем приравнять выражения, стоящие под знаками логарифмов.

14 – x = 5

x = 9

Сделайте проверку.

Ответ: 9

Решите самостоятельно:

Найдите корень уравнения log 5 (5 – x) = log 5 3.

Найдите корень уравнения: log 4 (x + 3) = log 4 (4x – 15).

Если log c a = log c b, то a = b

x + 3 = 4x – 15

3x = 18

x = 6

Сделайте проверку.

Ответ: 6

Найдите корень уравнения log 1/8 (13 – x) = – 2.

(1/8) –2 = 13 – x

8 2 = 13 – x

x = 13 – 64

x = – 51

Сделайте проверку.

Небольшое дополнение – здесь используется свойство

степени ().

Ответ: – 51

Решите самостоятельно:

Найдите корень уравнения: log 1/7 (7 – x) = – 2

Найдите корень уравнения log 2 (4 – x) = 2 log 2 5.

Преобразуем правую часть. воспользуемся свойством:

log a b m = m∙log a b

log 2 (4 – x) = log 2 5 2

Если log c a = log c b, то a = b

4 – x = 5 2

4 – x = 25

x = – 21

Сделайте проверку.

Ответ: – 21

Решите самостоятельно:

Найдите корень уравнения: log 5 (5 – x) = 2 log 5 3

Решите уравнение log 5 (x 2 + 4x) = log 5 (x 2 + 11)

Если log c a = log c b, то a = b

x 2 + 4x = x 2 + 11

4x = 11

x = 2,75

Сделайте проверку.

Ответ: 2,75

Решите самостоятельно:

Найдите корень уравнения log 5 (x 2 + x) = log 5 (x 2 + 10).

Решите уравнение log 2 (2 – x) = log 2 (2 – 3x) +1.

Необходимо с правой стороны уравнения получить выражение вида:

log 2 (……)

Представляем 1 как логарифм с основанием 2:

1 = log 2 2

log с (ab) = log с a + log с b

log 2 (2 – x) = log 2 (2 – 3x) + log 2 2

Получаем:

log 2 (2 – x) = log 2 2 (2 – 3x)

Если log c a = log c b, то a = b, значит

2 – x = 4 – 6x

5x = 2

x = 0,4

Сделайте проверку.

Ответ: 0,4

Решите самостоятельно: Далее необходимо решить квадратное уравнение. Кстати,

корни равны 6 и – 4.

Корень «– 4″ не является решением, так как основание логарифма должно быть больше нуля, а при » – 4″ оно равно « – 5». Решением является корень 6. Сделайте проверку.

Ответ: 6.

Решите самостоятельно:

Решите уравнение log x –5 49 = 2. Если уравнение имеет более одного корня, в ответе укажите меньший из них.

Как вы убедились, никаких сложных преобразований с логарифмическими уравнениями нет. Достаточно знать свойства логарифма и уметь применять их. В задачах ЕГЭ, связанных с преобразованием логарифмических выражений, выполняются более серьёзные преобразования и требуются более глубокие навыки в решении. Такие примеры мы рассмотрим, не пропустите! Успехов вам!!!

С уважением, Александр Крутицких.

P.S: Буду благодарен Вам, если расскажете о сайте в социальных сетях.

Заключительные видео из длинной серии уроков про решение логарифмических уравнений. В этот раз мы будем работать в первую очередь с ОДЗ логарифма — именно из-за неправильного учета (или вообще игнорирования) области определения возникает большинство ошибок при решении подобных задач.

В этом коротком видеоуроке мы разберем применение формул сложения и вычитания логарифмов, а также разберемся с дробно-рациональными уравнениями, с которыми у многих учеников также возникают проблемы.

О чем пойдет речь? Главная формула, с которой я хотел бы разобраться, выглядит так:

log a (f g ) = log a f + log a g

Это стандартный переход от произведения к сумме логарифмов и обратно. Вы наверняка знаете эту формулу с самого начала изучения логарифмов. Однако тут есть одна заминка.

До тех пор, пока в виде переменных a , f и g выступают обычные числа, никаких проблем не возникает. Данная формула работает прекрасно.

Однако, как только вместоf и g появляются функции, возникает проблема расширения или сужения области определения в зависимости от того, в какую сторону преобразовывать. Судите сами: в логарифме, записанном слева, область определения следующая:

fg > 0

А вот в сумме, записанной справа, область определения уже несколько иная:

f > 0

g > 0

Данный набор требований является более жестким, чем исходный. В первом случае нас устроит вариант f 0 выполняется).

Итак, при переходе от левой конструкции к правой возникает сужение области определения. Если же сначала у нас была сумма, а мы переписываем ее в виде произведения, то происходит расширение области определения.

Другими словами, в первом случае мы могли потерять корни, а во втором — получить лишние. Это необходимо учитывать при решении реальных логарифмических уравнений.

Итак, первая задача:

[Подпись к рисунку]

Слева мы видим сумму логарифмов по одному и тому же основанию. Следовательно, эти логарифмы можно сложить:

[Подпись к рисунку]

Как видите, справа мы заменил ноль по формуле:

a = log b b a

Давайте еще немного преобразуем наше уравнение:

log 4 (x − 5) 2 = log 4 1

Перед нами каноническая форма логарифмического уравнения, мы можем зачеркнуть знак log и приравнять аргументы:

(x − 5) 2 = 1

|x − 5| = 1

Обратите внимание: откуда взялся модуль? Напомню, что корень из точного квадрата равен именно модулю:

[Подпись к рисунку]

Затем решаем классическое уравнение с модулем:

|f | = g (g > 0) ⇒f = ±g

x − 5 = ±1 ⇒x 1 = 5 − 1 = 4; x 2 = 5 + 1 = 6

Вот два кандидат на ответ. Являются ли они решением исходного логарифмического уравнения? Нет, ни в коем случае!

Оставить все просто так и записать ответ мы не имеем права. Взгляните на тот шаг, когда мы заменяем сумму логарифмов одним логарифмом от произведения аргументов. Проблема в том, что в исходных выражениях у нас стоят функции. Следовательно, следует потребовать:

х(х − 5) > 0; (х − 5)/х > 0.

Когда же мы преобразовали произведение, получив точный квадрат, требования изменились:

(x − 5) 2 > 0

Когда это требование выполняется? Да практически всегда! За исключением того случая, когда х − 5 = 0. Т.е. неравенство сведется к одной выколотой точке:

х − 5 ≠ 0 ⇒ х ≠ 5

Как видим, произошло расширение области определения, о чем мы и говорили в самом начале урока. Следовательно, могут возникнуть и лишние корни.

Как же не допустить возникновения этих лишних корней? Очень просто: смотрим на наши полученные корни и сравниваем их с областью определения исходного уравнения. Давайте посчитаем:

х (х − 5) > 0

Решать будем с помощью метода интервалов:

х (х − 5) = 0 ⇒ х = 0; х = 5

Отмечаем полученные числа на прямой. Все точки выколотые, потому что неравенство строгое. Берем любое число, больше 5 и подставляем:

[Подпись к рисунку]

На интересуют промежутки (−∞; 0) ∪ (5; ∞). Если мы отметим наши корни на отрезке, то увидим, что х = 4 нас не устраивает, потому что этот корень лежит за пределами области определения исходного логарифмического уравнения.

Возвращаемся к совокупности, вычеркиваем корень х = 4 и записываем ответ: х = 6. Это уже окончательный ответ к исходному логарифмическому уравнению. Все, задача решена.

Переходим ко второму логарифмическому уравнению:

[Подпись к рисунку]

Решаем его. Заметим, что первое слагаемое представляет собой дробь, а второе — ту же самую дробь, но перевернутую. Не пугайтесь выражения lgx — это просто десятичный логарифм, мы можем записать:

lgx = log 10 x

Поскольку перед нами две перевернутые дроби, предлагаю ввести новую переменную:

[Подпись к рисунку]

Следовательно, наше уравнение может быть переписано следующим образом:

t + 1/t = 2;

t + 1/t − 2 = 0;

(t 2 − 2t + 1)/t = 0;

(t − 1) 2 /t = 0.

Как видим, в числителе дроби стоит точный квадрат. Дробь равна нулю, когда ее числитель равен нулю, а знаменатель отличен от нуля:

(t − 1) 2 = 0; t ≠ 0

Решаем первое уравнение:

t − 1 = 0;

t = 1.

Это значение удовлетворяет второму требованию. Следовательно, можно утверждать, что мы полностью решили наше уравнение, но только относительно переменной t . А теперь вспоминаем, что такое t :

[Подпись к рисунку]

Получили пропорцию:

lgx = 2 lgx + 1

2 lgx − lgx = −1

lgx = −1

Приводим это уравнение к канонической форме:

lgx = lg 10 −1

x = 10 −1 = 0,1

В итоге мы получили единственный корень, который, по идее, является решением исходного уравнения. Однако давайте все-таки подстрахуемся и выпишем область определения исходного уравнения:

[Подпись к рисунку]

Следовательно, наш корень удовлетворяет всем требованиям. Мы нашли решение исходного логарифмического уравнения. Ответ: x = 0,1. Задача решена.

Ключевой момент в сегодняшнем уроке один: при использовании формулы перехода от произведения к сумме и обратно обязательно учитывайте, что область определения может сужаться либо расширяться в зависимости от того, в какую сторону выполняется переход.

Как понять, что происходит: сужение или расширение? Очень просто. Если раньше функции были вместе, а теперь стали по отдельности, то произошло сужение области определения (потому что требований стало больше). Если же сначала функции стояли отдельно, а теперь — вместе, то происходит расширение области определения (на произведение накладывается меньше требований, чем на отдельные множители).

С учетом данного замечания хотел бы отметить, что второе логарифмическое уравнение вообще не требует данных преобразований, т. е. мы нигде не складываем и не перемножаем аргументы. Однако здесь я хотел бы обратить ваше внимание на другой замечательный прием, который позволяет существенно упростить решение. Речь идет о замене переменной.

Однако помните, что никакие замены не освобождает нас от области определения. Именно поэтому после того были найдены все корни, мы не поленились и вернулись к исходному уравнению, чтобы найти его ОДЗ.

Часто при замене переменной возникает обидная ошибка, когда ученики находят значение t и думают, что на этом решение закончено. Нет, ни в коем случае!

Когда вы нашли значение t , необходимо вернуться к исходному уравнению и посмотреть, что именно мы обозначали этой буквой. В результате нам предстоит решить еще одно уравнение, которое, впрочем, будет значительно проще исходного.

Именно в этом состоит смысл введения новой переменной. Мы разбиваем исходное уравнение на два промежуточных, каждое из которых решается существенно проще.

Как решать «вложенные» логарифмические уравнения

Сегодня мы продолжаем изучать логарифмические уравнения и разберем конструкции, когда один логарифм стоит под знаком другого логарифма. Оба уравнения мы будем решать с помощью канонической формы.

Сегодня мы продолжаем изучать логарифмические уравнения и разберем конструкции, когда один логарифм стоит под знаком другого. Оба уравнения мы будем решать с помощью канонической формы. Напомню, если у нас есть простейшее логарифмическое уравнение вида log a f (x ) = b , то для решения такого уравнения мы выполняем следующие шаги. В первую очередь, нам нужно заменить число b :

b = log a a b

Заметьте: a b — это аргумент. Точно так же в исходном уравнении аргументом является функция f (x ). Затем мы переписываем уравнение и получаем вот такую конструкцию:

log a f (x ) = log a a b

Уже затем мы можем выполнить третий шаг — избавится от знака логарифма и просто записать:

f (x ) = a b

В результате мы получим новое уравнение. При этом никаких ограничений на функцию f (x ) не накладывается. Например, на ее месте также может стоять логарифмическая функция. И тогда мы вновь получим логарифмическое уравнение, которое снова сведем к простейшему и решим через каноническую форму.

Впрочем, хватит лирики. Давайте решим настоящую задачу. Итак, задача № 1:

log 2 (1 + 3 log 2 x ) = 2

Как видим, перед нами простейшее логарифмическое уравнение. В роли f (x ) выступает конструкция 1 + 3 log 2 x , а в роли числа b выступает число 2 (в роли a также выступает двойка). Давайте перепишем эту двойку следующим образом:

Важно понимать, что первые две двойки пришли к нам из основания логарифма, т. е. если бы в исходном уравнении стояла 5, то мы бы получили, что 2 = log 5 5 2 . В общем, основание зависит исключительно от логарифма, который изначально дан в задаче. И в нашем случае это число 2.

Итак, переписываем наше логарифмическое уравнение с учетом того, что двойка, которая стоит справа, на самом деле тоже является логарифмом. Получим:

log 2 (1 + 3 log 2 x ) = log 2 4

Переходим к последнему шагу нашей схемы — избавляемся от канонической формы. Можно сказать, просто зачеркиваем знаки log. Однако с точки зрения математики «зачеркнуть log» невозможно — правильнее сказать, что мы просто просто приравниваем аргументы:

1 + 3 log 2 x = 4

Отсюда легко находится 3 log 2 x :

3 log 2 x = 3

log 2 x = 1

Мы вновь получили простейшее логарифмическое уравнение, давайте снова приведем его к канонической форме. Для этого нам необходимо провести следующие изменения:

1 = log 2 2 1 = log 2 2

Почему в основании именно двойка? Потому что в нашем каноническом уравнении слева стоит логарифм именно по основанию 2. Переписываем задачу с учетом этого факта:

log 2 x = log 2 2

Снова избавляемся от знака логарифма, т. е. просто приравниваем аргументы. Мы вправе это сделать, потому что основания одинаковые, и больше никаких дополнительных действий ни справа, ни слева не выполнялось:

Вот и все! Задача решена. Мы нашли решение логарифмического уравнения.

Обратите внимание! Хотя переменная х и стоит в аргументе (т. е. возникают требования к области определения), мы никаких дополнительных требований предъявлять не будем.

Как я уже говорил выше, данная проверка является избыточной, если переменная встречается лишь в одном аргументе лишь одного логарифма. В нашем случае х действительно стоит лишь в аргументе и лишь под одним знаком log. Следовательно, никаких дополнительных проверок выполнять не требуется.

Тем не менее, если вы не доверяете данному методу, то легко можете убедиться, что х = 2 действительно является корнем. Достаточно подставить это число в исходное уравнение.

Давайте перейдем ко второму уравнению, оно чуть интересней:

log 2 (log 1/2 (2x − 1) + log 2 4) = 1

Если обозначить выражение внутри большого логарифма функцией f (x ), получим простейшее логарифмическое уравнение, с которого мы начинали сегодняшний видеоурок. Следовательно, можно применить каноническую форму, для чего придется представить единицу в виде log 2 2 1 = log 2 2.

Переписываем наше большое уравнение:

log 2 (log 1/2 (2x − 1) + log 2 4) = log 2 2

Изваляемся от знака логарифма, приравнивая аргументы. Мы вправе это сделать, потому что и слева, и справа основания одинаковые. Кроме того, заметим, что log 2 4 = 2:

log 1/2 (2x − 1) + 2 = 2

log 1/2 (2x − 1) = 0

Перед нами снова простейшее логарифмическое уравнение вида log a f (x ) = b . Переходим к канонической форме, т. е. представляем ноль в виде log 1/2 (1/2)0 = log 1/2 1.

Переписываем наше уравнение и избавляемся от знака log, приравнивая аргументы:

log 1/2 (2x − 1) = log 1/2 1

2x − 1 = 1

Опять же мы сразу получили ответ. Никаких дополнительных проверок не требуется, потому что в исходном уравнении лишь один логарифм содержит функцию в аргументе.

Следовательно, никаких дополнительных проверок выполнять не требуется. Мы можем смело утверждать, что х = 1 является единственным корнем данного уравнения.

А вот если бы во втором логарифме вместо четверки стояла бы какая-то функция от х (либо 2х стояло бы не в аргументе, а в основании) — вот тогда потребовалось бы проверять область определения. Иначе велик шанс нарваться на лишние корни.

Откуда возникают такие лишние корни? Этот момент нужно очень четко понимать. Взгляните на исходные уравнения: везде функция х стоит под знаком логарифма. Следовательно, поскольку мы записали log 2 x , то автоматически выставляем требование х > 0. Иначе данная запись просто не имеет смысла.

Однако по мере решения логарифмического уравнения мы избавляемся от всех знаков log и получаем простенькие конструкции. Здесь уже никаких ограничений не выставляется, потому что линейная функция определена при любом значении х.

Именно эта проблема, когда итоговая функция определена везде и всегда, а исходная — отнюдь не везде и не всегда, и является причиной, по которой в решении логарифмических уравнениях очень часто возникают лишние корни.

Но повторю еще раз: такое происходить лишь в ситуации, когда функция стоит либо в нескольких логарифмах, либо в основании одного из них. В тех задачах, которые мы рассматриваем сегодня, проблем с расширением области определения в принципе не существует.

Случаи разного основания

Этот урок посвящен уже более сложным конструкциям. Логарифмы в сегодняшних уравнениях уже не будут решаться «напролом» — сначала потребуется выполнить некоторые преобразования.

Начинаем решение логарифмических уравнений с совершенно разными основаниями, которые не являются точными степенями друг друга. Пусть вас не пугают подобные задачи — решаются они ничуть не сложнее, чем самые простые конструкции, которые мы разбирали выше.

Но прежде, чем переходить непосредственно к задачам, напомню о формуле решения простейших логарифмических уравнений с помощью канонической формы. Рассмотрим задачу вот такого вида:

log a f (x ) = b

Важно, что функция f (x ) является именно функцией, а в роли чисел а и b должны выступать именно числа (без всяких переменных x ). Разумеется, буквально через минуту мы рассмотрим и такие случаи, когда вместо переменных а и b стоят функции, но сейчас не об этом.

Как мы помним, число b нужно заменить логарифмом по тому же самому основанию а, которое стоит слева. Это делается очень просто:

b = log a a b

Разумеется, под словом «любое число b » и «любое число а» подразумеваются такие значения, которые удовлетворяют области определения. В частности, в данном уравнении речь идет лишь основание a > 0 и a ≠ 1.

Однако данное требование выполняется автоматически, потому что в исходной задаче уже присутствует логарифм по основанию а — оно заведомо будет больше 0 и не равно 1. Поэтому продолжаем решение логарифмического уравнения:

log a f (x ) = log a a b

Подобная запись называется канонической формой. Ее удобство состоит в том, что мы сразу можем избавиться от знака log, приравняв аргументы:

f (x ) = a b

Именно этот прием мы сейчас будем использовать для решения логарифмических уравнений с переменным основанием. Итак, поехали!

log 2 (x 2 + 4x + 11) = log 0,5 0,125

Что дальше? Кто-то сейчас скажет, что нужно вычислить правый логарифм, либо свести их к одному основанию, либо что-то еще. И действительно, сейчас нужно привести оба основания к одному виду — либо 2, либо 0,5. Но давайте раз и навсегда усвоим следующее правило:

Если в логарифмическом уравнении присутствуют десятичные дроби, обязательно переведите эти дроби из десятичной записи в обычную. Такое преобразование может существенно упростить решение.

Подобный переход нужно выполнять сразу, еще до выполнения каких-либо действий и преобразований. Давайте посмотрим:

log 2 (x 2 + 4x + 11) = log 1 /2 1/8

Что нам дает такая запись? Мы можем 1/2 и 1/8 представить как степень с отрицательным показателем:


[Подпись к рисунку]

Перед нами каноническая форма. Приравниваем аргументы и получаем классическое квадратное уравнение:

x 2 + 4x + 11 = 8

x 2 + 4x + 3 = 0

Перед нами приведенное квадратное уравнение, которое легко решается с помощью формул Виета. Подобные выкладки в старших классах вы должны видеть буквально устно:

(х + 3)(х + 1) = 0

x 1 = −3

x 2 = −1

Вот и все! Исходное логарифмическое уравнение решено. Мы получили два корня.

Напомню, что определять область определения в данном случае не требуется, поскольку функция с переменной х присутствует лишь в одном аргументе. Поэтому область определения выполняется автоматически.

Итак, первое уравнение решено. Переходим ко второму:

log 0,5 (5x 2 + 9x + 2) = log 3 1/9

log 1/2 (5x 2 + 9x + 2) = log 3 9 −1

А теперь заметим, что аргумент первого логарифма тоже можно записать в виде степени с отрицательным показателем: 1/2 = 2 −1 . Затем можно вынести степени с обеих сторон уравнения и разделить все на −1:

[Подпись к рисунку]

И вот сейчас мы выполнили очень важный шаг в решении логарифмического уравнения. Возможно, кто-то что-то не заметил, поэтому давайте я поясню.

Взгляните на наше уравнение: и слева, и справа стоит знак log, но слева стоит логарифм по основанию 2, а справа стоит логарифм по основанию 3. Тройка не является целой степенью двойки и, наоборот: нельзя записать, что 2 — это 3 в целой степени.

Следовательно, это логарифмы с разными основаниями, которые не сводятся друг к другу простым вынесением степеней. Единственный путь решения таких задач — избавиться от одного из этих логарифмов. В данном случае, поскольку мы пока рассматриваем довольно простые задачи, логарифм справа просто сосчитался, и мы получили простейшее уравнение — именно такое, о котором мы говорили в самом начале сегодняшнего урока.

Давайте представим число 2, которое стоит справа в виде log 2 2 2 = log 2 4. А затем избавимся от знака логарифма, после чего у нас остается просто квадратное уравнение:

log 2 (5x 2 + 9x + 2) = log 2 4

5x 2 + 9x + 2 = 4

5x 2 + 9x − 2 = 0

Перед нами обычное квадратное уравнение, однако оно не является приведенным, потому что коэффициент при x 2 отличен от единицы. Следовательно, решать мы его будем с помощью дискриминанта:

D = 81 − 4 5 (−2) = 81 + 40 = 121

x 1 = (−9 + 11)/10 = 2/10 = 1/5

x 2 = (−9 − 11)/10 = −2

Вот и все! Мы нашли оба корня, а значит, получили решение исходного логарифмического уравнения. Ведь в исходной задачи функция с переменной х присутствует лишь в одном аргументе. Следовательно, никаких дополнительных проверок на область определения не требуется — оба корня, которые мы нашли, заведомо отвечают всем возможным ограничениям.

На этом можно было бы закончить сегодняшний видеоурок, но в заключении я хотел бы сказать еще раз: обязательно переводите все десятичные дроби в обычные при решении логарифмических уравнений. В большинстве случаев это существенно упрощает их решение.

Редко, очень редко попадаются задачи, в которых избавление от десятичных дробей лишь усложняет выкладки. Однако в таких уравнениях, как правило, изначально видно, что избавляться от десятичных дробей не надо.

В большинстве остальных случаев (особенно если вы только начинаете тренироваться в решении логарифмических уравнений) смело избавляйтесь от десятичных дробей и переводите их в обычные. Потому что практика показывает, что таким образом вы значительно упростите последующее решение и выкладки.

Тонкости и хитрости решения

Сегодня мы переходим к более сложным задачам и будем решать логарифмическое уравнение, в основании которого стоит не число, а функция.

И пусть даже эта функция линейна — в схему решения придется внести небольшие изменения, смысл которых сводится к дополнительным требованиям, накладываемым на область определения логарифма.

Сложные задачи

Этот урок будет довольно длинным. В нем мы разберем два довольно серьезных логарифмических уравнения, при решении которых многие ученики допускают ошибки. За свою практику работы репетитором по математике я постоянно сталкивался с двумя видами ошибок:

  1. Возникновение лишних корней из-за расширения области определения логарифмов. Чтобы не допускать такие обидные ошибки, просто внимательно следите за каждым преобразованием;
  2. Потери корней из-за того, что ученик забыл рассмотреть некоторые «тонкие» случаи — именно на таких ситуациях мы сегодня и сосредоточимся.

Это последний урок, посвященный логарифмическим уравнениям. Он будет длинным, мы разберем сложные логарифмические уравнения. Устраивайтесь поудобней, заварите себе чай, и мы начинаем.

Первое уравнение выглядит вполне стандартно:

log x + 1 (x − 0,5) = log x − 0,5 (x + 1)

Сразу заметим, что оба логарифма являются перевернутыми копиями друг друга. Вспоминаем замечательную формулу:

log a b = 1/log b a

Однако у этой формулы есть ряд ограничений, которые возникают в том случае, если вместо чисел а и b стоят функции от переменной х:

b > 0

1 ≠ a > 0

Эти требования накладываются на основание логарифма. С другой стороны, в дроби от нас требуется 1 ≠ a > 0, поскольку не только переменная a стоит в аргументе логарифма (следовательно, a > 0), но и сам логарифм находится в знаменателе дроби. Но log b 1 = 0, а знаменатель должен быть отличным от нуля, поэтому a ≠ 1.

Итак, ограничения на переменную a сохраняется. Но что происходит с переменной b ? С одной стороны, из основания следует b > 0, с другой — переменная b ≠ 1, потому что основание логарифма должно быть отлично от 1. Итого из правой части формулы следует, что 1 ≠ b > 0.

Но вот беда: второе требование (b ≠ 1) отсутствует в первом неравенстве, посвященном левому логарифму. Другими словами, при выполнении данного преобразования мы должны отдельно проверить , что аргумент b отличен от единицы!

Вот давайте и проверим. Применим нашу формулу:

[Подпись к рисунку]

1 ≠ х − 0,5 > 0; 1 ≠ х + 1 > 0

Вот мы и получили, что уже из исходного логарифмического уравнения следует, что и а, и b должны быть больше 0 и не равны 1. Значит, мы спокойно можем переворачивать логарифмическое уравнение:

Предлагаю ввести новую переменную:

log x + 1 (x − 0,5) = t

В этом случае наша конструкция перепишется следующим образом:

(t 2 − 1)/t = 0

Заметим, что в числителе у нас стоит разность квадратов. Раскрываем разность квадратов по формуле сокращенного умножения:

(t − 1)(t + 1)/t = 0

Дробь равна нулю, когда ее числитель равен нулю, а знаменатель отличен от нуля. Но в числителе стоит произведение, поэтому приравниваем к нулю каждый множитель:

t 1 = 1;

t 2 = −1;

t ≠ 0.

Как видим, оба значения переменной t нас устраивают. Однако на этом решение не заканчивается, ведь нам требуется найти не t , а значение x . Возвращаемся к логарифму и получаем:

log x + 1 (x − 0,5) = 1;

log x + 1 (x − 0,5) = −1.

Давайте приведем каждое из этих уравнений к канонической форме:

log x + 1 (x − 0,5) = log x + 1 (x + 1) 1

log x + 1 (x − 0,5) = log x + 1 (x + 1) −1

Избавляемся от знака логарифма в первом случае и приравниваем аргументы:

х − 0,5 = х + 1;

х − х = 1 + 0,5;

Такое уравнение не имеет корней, следовательно, первое логарифмическое уравнение также не имеет корней. А вот со вторым уравнением все намного интересней:

(х − 0,5)/1 = 1/(х + 1)

Решаем пропорцию — получим:

(х − 0,5)(х + 1) = 1

Напоминаю, что при решении логарифмических уравнений гораздо удобней приводить все десятичные дроби обычные, поэтому давайте перепишем наше уравнение следующим образом:

(х − 1/2)(х + 1) = 1;

x 2 + x − 1/2x − 1/2 − 1 = 0;

x 2 + 1/2x − 3/2 = 0.

Перед нами приведенное квадратное уравнение, оно легко решается по формулам Виета:

(х + 3/2) (х − 1) = 0;

x 1 = −1,5;

x 2 = 1.

Получили два корня — они являются кандидатами на решение исходного логарифмического уравнения. Для того чтобы понять, какие корни действительно пойдут в ответ, давайте вернемся к исходной задаче. Сейчас мы проверим каждый из наших корней на предмет соответствия области определения:

1,5 ≠ х > 0,5; 0 ≠ х > −1.

Эти требования равносильны двойному неравенству:

1 ≠ х > 0,5

Отсюда сразу видим, что корень х = −1,5 нас не устраивает, а вот х = 1 вполне устраивает. Поэтому х = 1 — окончательное решение логарифмического уравнения.

Переходим ко второй задаче:

log x 25 + log 125 x 5 = log 25 x 625

На первый взгляд может показаться, что у всех логарифмов разные основания и разные аргументы. Что делать с такими конструкциями? В первую очередь заметим, что числа 25, 5 и 625 — это степени 5:

25 = 5 2 ; 625 = 5 4

А теперь воспользуемся замечательным свойством логарифма. Дело в том, что можно выносить степени из аргумента в виде множителей:

log a b n = n ∙ log a b

На данное преобразование также накладываются ограничения в том случае, когда на месте b стоит функция. Но у нас b — это просто число, и никаких дополнительных ограничений не возникает. Перепишем наше уравнение:

2 ∙ log x 5 + log 125 x 5 = 4 ∙ log 25 x 5

Получили уравнение с тремя слагаемыми, содержащими знак log. Причем аргументы всех трех логарифмов равны.

Самое время перевернуть логарифмы, чтобы привести их к одному основанию — 5. Поскольку в роли переменной b выступает константа, никаких изменений области определения не возникает. Просто переписываем:


[Подпись к рисунку]

Как и предполагалось, в знаменателе «вылезли» одни и те же логарифмы. Предлагаю выполнить замену переменной:

log 5 x = t

В этом случае наше уравнение будет переписано следующим образом:

Выпишем числитель и раскроем скобки:

2 (t + 3) (t + 2) + t (t + 2) − 4t (t + 3) = 2 (t 2 + 5t + 6) + t 2 + 2t − 4t 2 − 12t = 2t 2 + 10t + 12 + t 2 + 2t − 4t 2 − 12t = −t 2 + 12

Возвращаемся к нашей дроби. Числитель должен быть равен нулю:

[Подпись к рисунку]

А знаменатель — отличен от нуля:

t ≠ 0; t ≠ −3; t ≠ −2

Последние требования выполняются автоматически, поскольку все они «завязаны» на целые числа, а все ответы — иррациональные.

Итак, дробно-рациональное уравнение решено, значения переменной t найдены. Возвращаемся к решению логарифмического уравнения и вспоминаем, что такое t :

[Подпись к рисунку]

Приводим это уравнение к канонической форме, получим число с иррациональной степенью. Пусть это вас не смущает — даже такие аргументы можно приравнять:

[Подпись к рисунку]

У нас получилось два корня. Точнее, два кандидата в ответы — проверим их на соответствие области определения. Поскольку в основании логарифма стоит переменная х, потребуем следующее:

1 ≠ х > 0;

С тем же успехом утверждаем, что х ≠ 1/125, иначе основание второго логарифма обратится в единицу. Наконец, х ≠ 1/25 для третьего логарифма.

Итого мы получили четыре ограничения:

1 ≠ х > 0; х ≠ 1/125; х ≠ 1/25

А теперь вопрос: удовлетворяют ли наши корни указанным требованиям? Конечно удовлетворяют! Потому что 5 в любой степени будет больше нуля, и требование х > 0 выполняется автоматически.

С другой стороны, 1 = 5 0 , 1/25 = 5 −2 , 1/125 = 5 −3 , а это значит, что данные ограничения для наших корней (у которых, напомню, в показателе стоит иррациональное число) также выполнены, и оба ответа являются решениями задачи.

Итак, мы получили окончательный ответ. Ключевых моментов в данной задаче два:

  1. Будьте внимательны при перевороте логарифма, когда аргумент и основание меняются местами. Подобные преобразования накладывают лишние ограничения на область определения.
  2. Не бойтесь преобразовывать логарифмы: их можно не только переворачивать, но и раскрывать по формуле суммы и вообще менять по любым формулам, которые вы изучали при решении логарифмических выражений. Однако при этом всегда помните: некоторые преобразования расширяют область определения, а некоторые — сужают.

Логарифмические уравнения. От простого — к сложному.

Внимание!
К этой теме имеются дополнительные
материалы в Особом разделе 555.
Для тех, кто сильно «не очень…»
И для тех, кто «очень даже…»)

Что такое логарифмическое уравнение?

Это уравнение с логарифмами. Вот удивил, да?) Тогда уточню. Это уравнение, в котором неизвестные (иксы) и выражения с ними находятся внутри логарифмов. И только там! Это важно.

Вот вам примеры логарифмических уравнений :

log 3 х = log 3 9

log 3 (х 2 -3) = log 3 (2х)

log х+1 (х 2 +3х-7) = 2

lg 2 (x+1)+10 = 11lg(x+1)

Ну, вы поняли… )

Обратите внимание! Самые разнообразные выражения с иксами располагаются исключительно внутри логарифмов. Если, вдруг, в уравнении обнаружится икс где-нибудь снаружи , например:

log 2 х = 3+х,

это будет уже уравнение смешанного типа. Такие уравнения не имеют чётких правил решения. Мы их пока рассматривать не будем. Кстати, попадаются уравнения, где внутри логарифмов только числа . Например:

Что тут сказать? Повезло вам, если попалось такое! Логарифм с числами — это какое-то число. И всё. Достаточно знать свойства логарифмов, чтобы решить такое уравнение. Знания специальных правил, приёмов, приспособленных именно для решения логарифмических уравнений, здесь не требуется.

Итак, что такое логарифмическое уравнение — разобрались.

Как решать логарифмические уравнения?

Решение логарифмических уравнений — штука, вообще-то, не очень простая. Так и раздел у нас — на четвёрку… Требуется приличный запас знаний по всяким смежным темам. Кроме того, существует в этих уравнениях особая фишка. И фишка это настолько важная, что её смело можно назвать главной проблемой в решении логарифмических уравнений. Мы с этой проблемой в следующем уроке детально разберёмся.

А сейчас — не волнуйтесь. Мы пойдём правильным путём, от простого к сложному. На конкретных примерах. Главное, вникайте в простые вещи и не ленитесь ходить по ссылкам, я их не просто так поставил… И всё у вас получится. Обязательно.

Начнём с самых элементарных, простейших уравнений. Для их решения желательно иметь представление о логарифме, но не более того. Просто без понятия логарифма, браться за решение логарифмических уравнений — как-то и неловко даже… Очень смело, я бы сказал).

Простейшие логарифмические уравнения.

Это уравнения вида:

1. log 3 х = log 3 9

2. log 7 (2х-3) = log 7 х

3. log 7 (50х-1) = 2

Процесс решения любого логарифмического уравнения заключается в переходе от уравнения с логарифмами к уравнению без них. В простейших уравнениях этот переход осуществляется в один шаг. Потому и простейшие.)

И решаются такие логарифмические уравнения на удивление просто. Смотрите сами.

Решаем первый пример:

log 3 х = log 3 9

Для решения этого примера почти ничего знать и не надо, да… Чисто интуиция!) Что нам особо не нравится в этом примере? Что-что… Логарифмы не нравятся! Правильно. Вот и избавимся от них. Пристально смотрим на пример, и у нас возникает естественное желание… Прямо-таки непреодолимое! Взять и выкинуть логарифмы вообще. И, что радует, это можно сделать! Математика позволяет. Логарифмы исчезают, получается ответ:

Здорово, правда? Так можно (и нужно) делать всегда. Ликвидация логарифмов подобным образом — один из основных способов решения логарифмических уравнений и неравенств. В математике эта операция называется потенцирование. Есть, конечно, свои правила на такую ликвидацию, но их мало. Запоминаем:

Ликвидировать логарифмы безо всяких опасений можно, если у них:

а) одинаковые числовые основания

в) логарифмы слева-справа чистые (безо всяких коэффициентов) и находятся в гордом одиночестве.

Поясню последний пункт. В уравнении, скажем,

log 3 х = 2log 3 (3х-1)

убирать логарифмы нельзя. Двойка справа не позволяет. Коэффициент, понимаешь… В примере

log 3 х+log 3 (х+1) = log 3 (3+х)

тоже нельзя потенцировать уравнение. В левой части нет одинокого логарифма. Их там два.

Короче, убирать логарифмы можно, если уравнение выглядит так и только так:

log а (…..) = log а (…..)

В скобках, где многоточие, могут быть какие угодно выражения. Простые, суперсложные, всякие. Какие угодно. Важно то, что после ликвидации логарифмов у нас остаётся более простое уравнение. Предполагается, конечно, что решать линейные, квадратные, дробные, показательные и прочие уравнения без логарифмов вы уже умеете.)

Теперь легко можно решить второй пример:

log 7 (2х-3) = log 7 х

Собственно, в уме решается. Потенцируем, получаем:

Ну что, очень сложно?) Как видите, логарифмическая часть решения уравнения заключается только в ликвидации логарифмов… А дальше идёт решение оставшегося уравнения уже без них. Пустяшное дело.

Решаем третий пример:

log 7 (50х-1) = 2

Видим, что слева стоит логарифм:

Вспоминаем, что этот логарифм — какое-то число, в которое надо возвести основание (т.е. семь), чтобы получить подлогарифменное выражение, т.е. (50х-1).

Но это число равно двум! По уравнению. Стало быть:

Вот, в сущности, и всё. Логарифм исчез, осталось безобидное уравнение:

Мы решили это логарифмическое уравнение исходя только из смысла логарифма. Что, ликвидировать логарифмы всё-таки проще?) Согласен. Между прочим, если из двойки логарифм сделать, можно этот пример и через ликвидацию решить. Из любого числа можно логарифм сделать. Причём, такой, какой нам надо. Очень полезный приём в решении логарифмических уравнений и (особо!) неравенств.

Не умеете из числа логарифм делать!? Ничего страшного. В разделе 555 этот приём подробно описан. Можете освоить и применять его на полную катушку! Он здорово уменьшает количество ошибок.

Совершенно аналогично (по определению) решается и четвёртое уравнение:

Вот и все дела.

Подведём итоги этого урока. Мы рассмотрели на примерах решение простейших логарифмических уравнений. Это очень важно. И не только потому, что такие уравнения бывают на контрольных-экзаменах. Дело в том, что даже самые злые и замороченные уравнения обязательно сводятся к простейшим!

Собственно, простейшие уравнения — это финишная часть решения любых уравнений. И эту финишную часть надо понимать железно! И ещё. Обязательно дочитайте эту страничку до конца. Есть там сюрприз…)

Решаем теперь самостоятельно. Набиваем руку, так сказать…)

Найти корень (или сумму корней, если их несколько) уравнений:

ln(7х+2) = ln(5х+20)

log 2 (х 2 +32) = log 2 (12x)

log 16 (0,5х-1,5) = 0,25

log 0,2 (3х-1) = -3

ln(е 2 +2х-3) = 2

log 2 (14х) = log 2 7 + 2

Ответы (в беспорядке, разумеется): 42; 12; 9; 25; 7; 1,5; 2; 16.

Что, не всё получается? Бывает. Не горюйте! В разделе 555 решение всех этих примеров расписано понятно и подробно. Там уж точно разберётесь. Да ещё и полезные практические приёмы освоите.

Всё получилось!? Все примеры «одной левой»?) Поздравляю!

Пришло время открыть вам горькую правду. Успешное решение этих примеров вовсе не гарантирует успех в решении всех остальных логарифмических уравнений. Даже простейших, подобных этим. Увы.

Дело в том, что решение любого логарифмического уравнения (даже самого элементарного!) состоит из двух равноценных частей. Решение уравнения, и работа с ОДЗ. Одну часть — решение самого уравнения — мы освоили. Не так уж и трудно, верно?

Для этого урока я специально подобрал такие примеры, в которых ОДЗ никак на ответе не сказывается. Но не все такие добрые, как я, правда?…)

Посему надо обязательно освоить и другую часть. ОДЗ. Это и есть главная проблема в решении логарифмических уравнений. И не потому, что трудная — эта часть ещё проще первой. А потому, что про ОДЗ просто забывают. Или не знают. Или и то, и другое). И падают на ровном месте…

В следующем уроке мы расправимся с этой проблемой. Вот тогда можно будет уверенно решать любые несложные логарифмические уравнения и подбираться к вполне солидным заданиям.

Если Вам нравится этот сайт…

Кстати, у меня есть ещё парочка интересных сайтов для Вас.)

Можно потренироваться в решении примеров и узнать свой уровень. Тестирование с мгновенной проверкой. Учимся — с интересом!)

можно познакомиться с функциями и производными.

Соблюдение Вашей конфиденциальности важно для нас. По этой причине, мы разработали Политику Конфиденциальности, которая описывает, как мы используем и храним Вашу информацию. Пожалуйста, ознакомьтесь с нашими правилами соблюдения конфиденциальности и сообщите нам, если у вас возникнут какие-либо вопросы.

Сбор и использование персональной информации

Под персональной информацией понимаются данные, которые могут быть использованы для идентификации определенного лица либо связи с ним.

От вас может быть запрошено предоставление вашей персональной информации в любой момент, когда вы связываетесь с нами.

Ниже приведены некоторые примеры типов персональной информации, которую мы можем собирать, и как мы можем использовать такую информацию.

Какую персональную информацию мы собираем:

  • Когда вы оставляете заявку на сайте, мы можем собирать различную информацию, включая ваши имя, номер телефона, адрес электронной почты и т.д.

Как мы используем вашу персональную информацию:

  • Собираемая нами персональная информация позволяет нам связываться с вами и сообщать об уникальных предложениях, акциях и других мероприятиях и ближайших событиях.
  • Время от времени, мы можем использовать вашу персональную информацию для отправки важных уведомлений и сообщений.
  • Мы также можем использовать персональную информацию для внутренних целей, таких как проведения аудита, анализа данных и различных исследований в целях улучшения услуг предоставляемых нами и предоставления Вам рекомендаций относительно наших услуг.
  • Если вы принимаете участие в розыгрыше призов, конкурсе или сходном стимулирующем мероприятии, мы можем использовать предоставляемую вами информацию для управления такими программами.

Раскрытие информации третьим лицам

Мы не раскрываем полученную от Вас информацию третьим лицам.

Исключения:

  • В случае если необходимо — в соответствии с законом, судебным порядком, в судебном разбирательстве, и/или на основании публичных запросов или запросов от государственных органов на территории РФ — раскрыть вашу персональную информацию. Мы также можем раскрывать информацию о вас если мы определим, что такое раскрытие необходимо или уместно в целях безопасности, поддержания правопорядка, или иных общественно важных случаях.
  • В случае реорганизации, слияния или продажи мы можем передать собираемую нами персональную информацию соответствующему третьему лицу – правопреемнику.

Защита персональной информации

Мы предпринимаем меры предосторожности — включая административные, технические и физические — для защиты вашей персональной информации от утраты, кражи, и недобросовестного использования, а также от несанкционированного доступа, раскрытия, изменения и уничтожения.

Соблюдение вашей конфиденциальности на уровне компании

Для того чтобы убедиться, что ваша персональная информация находится в безопасности, мы доводим нормы соблюдения конфиденциальности и безопасности до наших сотрудников, и строго следим за исполнением мер соблюдения конфиденциальности.

С уравнениями мы все знакомы с начальных классов. Еще там мы учились решать самые простые примеры, и надо признать, что они находят свое применение даже в высшей математике. С уравнениями все просто, в том числи и с квадратными. Если у вас проблемы с этой темой, настоятельно рекомендуем вам повторить ее.

Логарифмы вы, вероятно, тоже уже прошли. Тем не менее, считаем важным рассказать, что это для тех, кто еще не знает. Логарифм приравнивается к степени, в которую нужно возвести основание, чтобы получилось число, стоящее справа от знака логарифма. Приведем пример, исходя из которого, вам все станет ясно.

Если вы возведете 3 в четвертую степень получится 81. Теперь подставьте по аналогии числа, и поймете окончательно, как решаются логарифмы. Теперь осталось лишь совместить два рассмотренных понятия. Изначально ситуация кажется чрезвычайно сложной, но при ближайшем рассмотрении весе становится на свои места. Мы уверены, что после этой короткой статьи у вас не будет проблем в этой части ЕГЭ.

Сегодня выделяют множество способов решения подобных конструкций. Мы расскажем о самых простых, эффективных и наиболее применимых в случае заданий ЕГЭ. Решение логарифмических уравнений должно начинаться с самого простого примера. Простейшие логарифмические уравнения состоят из функции и одной переменной в ней.

Важно учесть, что x находится внутри аргумента. A и b должны быть числами. В таком случае вы можете попросту выразить функцию через число в степени. Выглядит это следующим образом.

Разумеется, решение логарифмического уравнения таким методом приведет вас к верному ответу. Ног проблема подавляющего большинства учеников в этом случае заключается в том, что они не понимают, что и откуда берется. В результате приходится мириться с ошибками и не получать желаемых баллов. Самой обидной ошибкой будет, если вы перепутаете буквы местами. Чтобы решить уравнение этим способом, нужно зазубрить эту стандартную школьную формулу, потому что понять ее сложно.

Чтобы было проще, можно прибегнуть к другому способу – канонической форме. Идея крайне проста. Снова обратите внимание на задачу. Помните, что буква a – число, а не функция или переменная. A не равно одному и больше нуля. На b никаких ограничений не действует. Теперь из всех формул вспоминаем одну. B можно выразить следующим образом.

Из этого следует, что все исходные уравнения с логарифмами можно представить в виде:

Теперь мы можем отбросить логарифмы. Получится простая конструкция, которую мы уже видели ранее.

Удобство данной формулы заключается в том, что ее можно применять в самых разных случаях, а не только для самых простых конструкций.

Не переживайте насчет ООФ!

Многие опытные математики заметят, что мы не уделили внимание области определения. Сводится правило к тому, что F(x) обязательно больше 0. Нет, мы не упустили этот момент. Сейчас мы говорим об еще одном серьезном преимуществе канонической формы.

Лишних корней здесь не возникнет. Если переменная будет встречаться лишь в одном месте, то область определения не является необходимостью. Она выполняется автоматически. Чтобы убедиться в данном суждении, займитесь решением нескольких простых примеров.

Как решать логарифмические уравнения с разными основаниями

Это уже сложные логарифмические уравнения, и подход к их решению должен быть особым. Здесь редко получается ограничиться пресловутой канонической формой. Начнем наш подробный рассказ. Мы имеем следующую конструкцию.

Обратите внимание на дробь. В ней находится логарифм. Если вы увидите такое в задании, стоит вспомнить один интересный прием.

Что это значит? Каждый логарифм можно представить в виде частного двух логарифмов с удобным основанием. И у данной формулы есть частный случай, который применим с этим примером (имеем ввиду, если c=b).

Именно такую дробь мы и видим в нашем примере. Таким образом.

По сути, перевернули дробь и получили более удобное выражение. Запомните этот алгоритм!

Теперь нужно, что логарифмическое уравнение не содержало разных оснований. Представим основание дробью.

В математике есть правило, исходя из которого, можно вынести степень из основания. Получается следующая конструкция.

Казалось бы, что мешает теперь превратить наше выражение в каноническую форму и элементарно решить ее? Не все так просто. Дробей перед логарифмом быть не должно. Исправляем эту ситуацию! Дробь разрешается выносить в качестве степени.

Соответственно.

Если основания одинаковые, мы можем убрать логарифмы и приравнять сами выражения. Так ситуация станет в разы проще, чем была. Останется элементарное уравнение, которое каждый из нас умел решать еще в 8 или даже в 7 классе. Расчеты вы сможете произвести сами.

Мы получили единственно верный корень этого логарифмического уравнения. Примеры решения логарифмического уравнения достаточно просты, не так ли? Теперь и у вас получится самостоятельно разобраться даже с самыми сложными задачами для подготовки и сдачи ЕГЭ.

Что в итоге?

В случае с любыми логарифмическими уравнениями мы исходим из одного очень важного правила. Необходимо действовать так, чтобы привести выражение к максимально простому виду. В таком случае у вас будет больше шансов не просто решить задание правильно, но еще и сделать это максимально простым и логичным путем. Именно так всегда действуют математики.

Настоятельно не рекомендуем вам искать сложных путей, особенно в этом случае. Запомните несколько простых правил, которые позволят преобразовать любое выражение. К примеру, привести два или три логарифма к одному основанию или вывести степень из основания и выиграть на этом.

Также стоит помнить о том, что в решении логарифмических уравнений необходимо постоянно тренироваться. Постепенно вы будете переходить ко все более сложным конструкциям, а это приведет вас к уверенному решению всех вариантов задач на ЕГЭ. Готовьтесь к экзаменам заблаговременно, и удачи вам!

4.6: Экспоненциальные и логарифмические уравнения

  1. Последнее обновление
  2. Сохранить как PDF
  • Идентификатор страницы
    34907
  • Цели обучения

    • Решить экспоненциальные уравнения, переписав их с общим основанием или переписав в логарифмической форме.
    • Решите логарифмические уравнения, переписав их в экспоненциальной форме или используя свойство логарифмов «один к одному».

    В 1859 году австралийский землевладелец по имени Томас Остин выпустил \(24\) кроликов в дикую природу для охоты. Поскольку в Австралии было мало хищников и достаточно еды, популяция кроликов резко возросла. Менее чем за десять лет популяция кроликов исчислялась миллионами. Неконтролируемый рост популяции, как у диких кроликов в Австралии, можно смоделировать с помощью экспоненциальных функций. Уравнения, полученные из этих экспоненциальных функций, можно решать для анализа и прогнозирования экспоненциального роста. В этом разделе мы изучим методы решения экспоненциальных функций. 9Т\) тогда и только тогда, когда \(S=T\).

    Другими словами, когда экспоненциальное уравнение имеет одинаковое основание с каждой стороны, показатели степени должны быть равны. Это также применимо, когда показатели степени являются алгебраическими выражениями. T\). 9х=\sqrt{5}\).

    Ответить

    \(х=\dfrac{1}{2}\)

    Все ли показательные уравнения имеют решение? Как мы можем сказать, что решения нет?

    Нет. Напомним, что диапазон экспоненциальной функции всегда положителен. В процессе решения показательного уравнения, если полученное уравнение представляет собой показательное выражение, не равное положительному числу, для этого уравнения нет решения.

    9х=-100\).

    Ответить

    Уравнение не имеет решения.

    Переписать в логарифмической форме 

    Иногда члены экспоненциального уравнения невозможно переписать с общим основанием. В этих случаях мы решаем, логарифмируя каждую сторону. Напомним, поскольку \(\log(a)=\log(b)\) эквивалентно \(a=b\), мы можем применять логарифмы с одинаковым основанием к обеим частям экспоненциального уравнения.

     Как: решить показательное уравнение, в котором не удается найти общее основание 9x \qquad &&\text{Взять ln обеих сторон}\\
    (x+2)\ln5&= x\ln4 \qquad &&\text{степенное правило для логарифмов}\\
    x\ln5+2\ln5&= x \ln4 \qquad &&\text{Используйте распределительный закон}\\
    x\ln5-x\ln4&= -2\ln5 \qquad &&\text{Получите термы, содержащие x с одной стороны, термы без x с другой}\ \
    x(\ln5-\ln4)&= -2\ln5 \qquad &&\text{В левой части вынесите x}\\
    x\ln \left (\frac{5}{4} \right )&= \ln \left (\frac{1}{25} \right ) \qquad &&\text{Степенные и частные правила для логарифмов}\\ 9Икс\)?

    Да. {kt}\), найдите \(t\). 9x \qquad &&\text{Переписать как логарифмическое уравнение}\\
    x &= \log_\ce{4/5} (25) \qquad &&\text{Теперь используйте правило изменения базы}\\
    x & = \dfrac{\ln 25}{\ln \ce{4/5}} \ приблизительно -14,4251
    \end{align*}\]

    Экспоненциальные уравнения, квадратичные по форме

    Посторонние решения

    Иногда методы используемое для решения уравнения, ввести лишнее решение , которое является решением, правильным алгебраически, но не удовлетворяющим условиям исходного уравнения. Одна из таких ситуаций возникает при решении, когда обе части уравнения логарифмируются. В таких случаях помните, что аргумент логарифма должен быть положительным. Если значение аргумента логарифма отрицательное, вывод невозможен. 9x&= 8 \\
    x &= \ln 8 \\
    \end{array} } && { \begin{array} {l}
    \text{Используйте свойство нулевого фактора для нахождения } x \\
    \text {Переписать в виде журнала } \\
    \text{Отклонить уравнение, в котором степень равна отрицательному числу }
    \end{array} } \\
     \end{align*}\]

    Анализ

    Когда мы планируем использовать факторинг для решения проблемы, мы всегда получаем ноль с одной стороны уравнения, потому что ноль обладает уникальным свойством: когда произведение равно нулю, один или оба множителя должны быть равны нулю. y=x\). Мы можем использовать этот факт вместе с правилами логарифмирования для решения логарифмических уравнений, где аргументом является алгебраическое выражение. 93\\ 18&= 6x \qquad &&\text{Прибавьте 10 к обеим сторонам}\\ x&= 3 \qquad &&\text{Разделите на 6, затем проверьте решение!} \end{align*}\]

    Когда угодно решая уравнение с логарифмами, всегда необходимо проверять, что решение находится в области определения исходного уравнения. Если это не так, оно должно быть отвергнуто как решение.

    ИСПОЛЬЗУЙТЕ ОПРЕДЕЛЕНИЕ ЛОГАРИФМА ДЛЯ РЕШЕНИЯ ЛОГАРИТМИЧЕСКИХ УРАВНЕНИЙ

    Для любого алгебраического выражения \(S\) и действительных чисел \(b\) и \(c\), где \(b>0\), \(b ≠1\), 95−1\)

    Использование свойства «один к одному» логарифмов

    Как и в случае показательных уравнений, мы можем использовать свойство «один к одному» для решения логарифмических уравнений. Свойство взаимно однозначности логарифмических функций говорит нам, что для любых действительных чисел \(S>0\), \(T>0\) и любого положительного действительного числа \(b\), где \(b≠1 \),

    \({\log}_bS={\log}_bT\) тогда и только тогда, когда \(S=T\).

    Например,

    Если \({\log}_2(x−1)={\log}_2(8)\), то \(x−1=8\).

    Итак, если \(x−1=8\), то мы можем найти \(x\), и мы получим \(x=9\). Чтобы проверить, мы можем подставить \(x=9\) в исходное уравнение: \({\log}_2(9−1)={\log}_2(8)=3.\) Другими словами, когда логарифмическое уравнение имеет одинаковое основание с каждой стороны, аргументы должны быть равны. Это также применимо, когда аргументы являются алгебраическими выражениями. Поэтому, когда у нас есть уравнение с логарифмами одинакового основания на каждой стороне, мы можем использовать правила логарифмирования, чтобы переписать каждую сторону как один логарифм. Затем мы используем тот факт, что логарифмические функции являются взаимно однозначными, чтобы установить аргументы равными друг другу и найти неизвестное.

    Например, рассмотрим уравнение \(\log(3x−2)−\log(2)=\log(x+4)\). Чтобы решить это уравнение, мы можем использовать правила логарифмирования, чтобы переписать левую часть как одиночный логарифм, а затем применить свойство «один к одному» для решения для \(x\):

    \[\begin{align* } \log(3x-2)-\log(2)&= \log(x+4)\\ \log\left (\dfrac{3x-2}{2} \right )&= \log(x+ 4) \qquad &&\text{Применить правило отношения логарифмов}\\ \dfrac{3x-2}{2}&= x+4 \qquad &&\text{Применить свойство один к одному логарифма}\\ 3x-2&= 2x+8 \qquad &&\text{Умножьте обе части уравнения на 2}\\ x&= 10 \qquad &&\text{Вычтите 2x и прибавьте 2} \end{align*}\]

    Чтобы проверить результат, подставьте \(x=10\) в \(\log(3x−2)−\log(2)=\log(x+4)\).

    \[\begin{align*} \log(3(10)-2)-\log(2)&= \log((10)+4) \\ \log(28)-\log(2) &= \log(14)\\ \log \left (\dfrac{28}{2} \right )&= \log(14) \qquad \text{Проверка решения} \end{align*}\]

    ИСПОЛЬЗОВАТЬ СВОЙСТВО ОДНОЗНАЧНОСТИ ЛОГАРИФМОВ ДЛЯ РЕШЕНИЯ ЛОГАРИТМИЧЕСКИХ УРАВНЕНИЙ

    Для любых алгебраических выражений \(S\) и \(T\) и любого положительного действительного числа \(b\), где \(b≠1\ ),

    \[\begin{align*} {\log}_bS={\log}_bT \quad \text{ тогда и только тогда, когда } \quad S=T \end{align*}\]

    Обратите внимание, при решении уравнение с логарифмами, всегда проверяйте, правильный ли ответ или это постороннее решение.

     Как: Имея уравнение, содержащее логарифмы, решить его, используя свойство однозначности _bS={\log}_bT\). 9с=S\). Свойство один к одному для логарифмических функций Для любых алгебраических выражений \(S\) и \(T\) и любого положительного действительного числа \(b\), где \(b≠1\),
    \({\log}_bS={\log}_bT \) тогда и только тогда, когда \(S=T\).

    Ключевые понятия

    • Мы можем решить многие экспоненциальные уравнения, используя правила экспонент, чтобы представить каждую часть как степень с одним и тем же основанием. Затем мы используем тот факт, что экспоненциальные функции являются взаимно однозначными, чтобы установить показатели степени равными друг другу и найти неизвестное.
    • Когда нам дано экспоненциальное уравнение, в котором основания явным образом показаны как равные, приравняйте показатели степени друг к другу и найдите неизвестное.
    • Когда нам дано экспоненциальное уравнение, в котором основания равны , а не , явным образом показанные равными, перепишем каждую часть уравнения как степени одного и того же основания, затем приравняем показатели степени и найдем неизвестное.
    • Если экспоненциальное уравнение нельзя переписать с общим основанием, решите его путем логарифмирования каждой стороны. 9c=S\) и найти неизвестное.
    • Когда задано уравнение формы \({\log}_bS={\log}_bT\), где \(S\) и \(T\) — алгебраические выражения, мы можем использовать свойство однозначности логарифмов для решения уравнения \(S=T\) относительно неизвестного. Убедитесь, что ответы не делают исходные аргументы журнала нулевыми или отрицательными.

    Авторы


    1. Наверх
      • Была ли эта статья полезной?
      1. Тип изделия
        Раздел или Страница
        Лицензия
        СС BY
        Показать страницу TOC
        да
      2. Теги
        1. определение логарифма
        2. однозначное свойство экспоненциальных функций
        3. однозначное свойство для логарифмических функций
        4. источник[1]-math-1357
        5. источник-математика-1357

      БиоМатематика: логарифмические функции

      В этом разделе мы научимся решать уравнения, содержащие показательную или логарифмическая функция. Мы начнем с рассмотрения двух простых уравнений

      (1) 5 e x = 11,

      (2) log (2 x ) = 3,

      Чтобы решить (1) и (2), нам нужно найти значение (я) x , для которых уравнения истинное утверждение. Важно понимать, что (1) и (2) НЕ верны для всех значения x . Начнем с уравнения (1).

      Решение уравнения 1:

      5 e x = 11

      Как и при решении любого другого уравнения, наша цель — выделить член, включающий x . Здесь мы начинаем с деления обеих частей уравнения на 5

      .

      Теперь, когда член e x сам по себе находится на одной стороне, мы должны «отменить» экспоненциальную чтобы освободить х . Поскольку наша экспонента имеет основание e , мы берем логарифм по основанию e обе стороны (т.е. натуральный логарифм обеих сторон),

      Таким образом, решение уравнения (1) равно

      Решение уравнения 2:

      log (2 x ) = 3


      В этом уравнении член, включающий x уже выделено в одну часть уравнения. Поэтому мы начнем с «отмены» логарифмической функции по основанию 10 путем возведения в степень обоих стороны уравнения с основанием 10,

      10 log (2 x ) = 2 x = 10 3 = 1000.

      Следовательно, мы находим решение уравнения (2) равным

      Сейчас попробуем решить еще несколько сложных уравнений.

      Хотя эти уравнения могут показаться более сложными, наша цель остается прежней: решите значение (я) x , которые делают утверждения уравнений верными.

      Решение экспоненциальных уравнений с основанием, которого нет в вашем калькуляторе

      Решение уравнения 3 :

      6 · 2 9+ x = 30

      Начнем с выделения термина, включающего x ,

      Чтобы найти x , мы должны отменить экспоненту. Несмотря на то, что экспонента по основанию 2, мы будем использовать десятичный логарифм, так как его будет легче получить примерный ответ на нашем калькуляторе,

      Используя свойства логарифмов, мы упрощаем приведенное выше уравнение как,

      (9 + х ) логарифм (2) = логарифм (5).

      Решение x дает,


      Вы всегда можете проверить свой ответ, подставив точный ответ, который вы нашли обратно в исходное уравнение.

      Решение уравнения 4:

      3 log 6 (4 + 3 x ) — 2 = 6

      Начнем с выделения члена, включающего х ,

      Чтобы найти x , нам нужно освободить x из логарифма по основанию 6. Для этого возводим обе части в степень с основанием 6,

      .

      Решение для x у нас есть,

      Решение уравнений, имеющих экспоненты с разными основаниями

      Решение уравнения 5:

      3 7−2 x = 4 3+ x

      В этом уравнении экспоненты имеют разные основания. Мы возьмем обычный (по основанию 10) логарифм обеих сторон, чтобы отменить эти экспоненты, и использовать свойства логарифмов,

      Перенос терминов размером x в одну сторону и всех остальных терминов в другую,

      Факторизация выходов x ,

      Теперь мы можем решить уравнение

      Решение уравнений с кратными логарифмами

      Решение уравнения 6:

      2 лог. 3 (2x) = лог. 3 (4) + лог. 3 (3 + 2x)

      В этом уравнении следует обратить внимание на две вещи: обе части имеют логарифмы, а одна сторона имеет несколько логарифмов. При работе с несколькими логарифмами на одна сторона, проще всего объединить их в один логарифм, используя в свойства логарифмов. Однако при этом вам необходимо проверить свои решения. когда вы закончите решать проблему, потому что объединение логарифмов может привести к посторонним решениям.

      Начнем с упрощения обеих сторон, используя свойства логарифмы,

      Возведение в степень обеих сторон с основанием 3 дает,

      Чтобы решить для x , мы должны решить квадратное уравнение,

      4 х 2 — 8 х — 12 = 0,

      Используя квадратичную формулу, решения даются как,

      Таким образом, мы находим решения равными x = 3 и x = −1. Поскольку мы объединили логарифмы, мы должны проверить эти решения в исходном уравнении. Проверка
      х
      = 3 дает,

      2 log 3 (2 · 3) = 2 log 3 (6) ≈ 3,26,

      с левой стороны и

      log 3 (4) + log 3 (3 + 2 · 3) = log 3 (4) + log 3 (9) ≈ 3,26,

      с правой стороны. Эти решения проверяют.

      Однако решение x = −1 не проверяет. Если мы попытаемся подставить
      x = −1 в исходное уравнение, мы бы,

      2 журнал 3 (2 · −1),

      с левой стороны. Конечно, логарифм отрицательного числа не определен. на вещественной прямой, и мы заключаем, что x = −1 является посторонним решением.

      Следовательно, мы заключаем, что решение уравнения (6) равно x = 3,

      .

       

      *****

      Теперь попробуйте решить некоторые практические задачи с использованием логарифмов и решением логарифмических уравнений.

      Проблемы

       

      Решение логарифмических уравнений | Brilliant Math & Science Wiki

      Для решения многих уравнений с логарифмами достаточно просто использовать определение log⁡x \log x logx для исключения логарифмов из уравнения и преобразования его в полиномиальное или экспоненциальное уравнение.

      Найдите x x x, если log⁡2(3x+1)=4 \log_2(3x+1) = 4 log2​(3x+1)=4.


      По определению логарифма, 94 = 16 24 = 16. Более сложные логарифмические уравнения часто упрощают, возводя в степень обе части.

      Если log⁡8м+log⁡816=23\log_{8}м + \log_{8}\frac{1}{6}=\frac{2}{3}log8​m+log8​61​= 32​, то что такое м?м?м?

      Другие уравнения можно упростить, используя другие свойства логарифмов. Одна из возникающих трудностей заключается в том, что устранение логарифмов и решение полученного уравнения могут привести к ложным решениям. Эти решения нарушают принцип, согласно которому аргумент логарифмической функции всегда должен быть положительным. Обычно разумно проверять решения, вставляя их в исходное уравнение и убеждаясь, что обе стороны определены. 92-18х-45 &=0\\ (х-15)(х-3) &= 0. \end{выровнено}log3​(x−12)+log3​(x−6)log3​((x−12)(x−6))(x−12)(x−6)x2−18x−45( х-15)(х-3)​=3=3=27=0=0.​

      Это дает два потенциальных решения x=3,x=15 x=3, x=15x=3,x=15. Но обратите внимание, что x=3 x = 3 x=3 не является реальным решением, так как log⁡3(3−12) \log_3(3-12) log3​(3−12) не определено. Единственное актуальное решение: x=15 x=15 x=15. □_\квадрат□​

      Шаг в приведенном выше решении, который не был обратимым, был первым: хотя (x−12)(x−6) (x-12)(x-6)(x−12)(x−6) равно положительно, если x=3 x =3x=3, x−12 x-12 x−12 и x−6 x-6x−6 сами по себе не положительны.

      1 -1 2 0

      Найти все действительные решения xxx для

      3log⁡2(x)−1=log⁡2(32x−1).3\log_2(x) — 1 = \log_2\left(\frac32 x-1\right ).3log2​(x)−1=log2​(23​x−1).

      Введите ответ как сумму всех таких x x x.

      Более сложные логарифмические уравнения часто включают более одного основания. Это может помочь ввести неизвестные, чтобы сначала найти логарифмы. Другое полезное тождество: log⁡x(y)=log⁡z(y)log⁡z(x) \log_x(y) = \frac{\log_z(y)}{\log_z(x)} logx​(y) =logz​(x)logz​(y)​, тем более что z zz можно выбрать любым, что упрощает задачу. 92=б а2=б.


      Перепишите это как

      log⁡(10)log⁡(a)+log⁡(100)log⁡(b)=log⁡(1000000)log⁡(ab),\frac{\log(10)}{\log(a)} + \frac{\log(100)}{\log(b)} = \frac{\log(1000000)}{\log(ab)},log(a)log(10)​+log(b)log (100)​=log(ab)log(1000000)​,

      , где все журналы с основанием 10 10 10. Это упрощается до

      .

      1log⁡(a)+2log⁡(b)=6log⁡(ab).\frac1{\log(a)}+\frac2{\log(b)} = \frac6{\log(ab)}.log (a)1​+log(b)2​=log(ab)6​.

      Пусть m=log⁡(a) m = \log(a) m=log(a) и n=log⁡(b) n = \log(b) n=log(b). Тогда log⁡(ab)=m+n\log(ab) = m+nlog(ab)=m+n, поэтому 92=б а2=б. □_\квадрат □​

      Сумма всех (положительных) решений уравнения \log_x {512}log16​x+logx​16=log512​x+logx​512

      можно записать как ab \frac{a}{b} ba​, где aaa и bbb — взаимно простые положительные целые числа. Каковы последние три цифры числа a+ba+ba+b?

      Уравнения с показателями степени часто можно упростить, взяв логарифмы:

      Решите для х х х, если 93}{25}?xlog5​x=25×3​?

      Логарифмы также могут упростить вычисления в некоторых практических ситуациях. Например, логарифмы с основанием 101010 дают информацию о количестве десятичных цифр в числе.

      Учитывая, что

      log⁡10(2)=0,3010299…log⁡10(3)=0,4771212…log⁡10(7)=0,8450980…,\begin{выровнено} \log_{10}(2) &= 0,3010299\ldots \\ \log_{10}(3) &= 0,4771212\ldots \\ \log_{10}(7) &= 0,8450980\ldots, \end{выровнено}log10​(2)log10​(3)log10​(7)​=0,301029{2}}992.

      Примечание : Вы можете использовать тот факт, что log⁡103=0,4771\log_{10} 3 = 0,4771log10​3=0,4771 исправить до 4 знаков после запятой.

      Решение логарифмических уравнений: пошаговое руководство

      12 ноября 2021 г.

      Что такое логарифмические уравнения?

      Изучаете ли вы логарифмы на уроках математики в старшей школе или готовитесь к стандартизированному тесту, такому как ACT или SAT, эта конкретная концепция часто является одной из самых сложных, с которыми вы сталкиваетесь в математике. Логарифм — это обратная функция возведенного в степень числа. Это запутанное определение означает, что логарифм — это запись, обратная тому, что вы обычно видите при вычислении показателя степени. Сочетание сложности концепции с математической сложностью вычислений вызывает у учащихся такие проблемы с логарифмами.

      Поскольку логарифмы — это просто еще один способ записи показателей степени, вам следует начать с изучения основ того, что такое показатели степени и как они работают. Показатель степени показывает, сколько раз число (или переменная, например x ) умножается само на себя. В простейшей форме это означает, что, например, 5 3 = 5 * 5 * 5. Хотя этот конкретный пример даст вам ответ в виде вещественного числа (125), часто учащиеся сталкиваются с показателями степени в уравнениях алгебры с переменными. , например y = 2 x 4 – 6 x 2 – 56.

      Экспоненты построены таким образом, что вы знаете число или переменную, которую вы умножаете само на себя, также известную как основание, и вы знать, сколько раз вы умножаете его на себя, показатель степени. Ваша цель — найти результат этого вычисления, другими словами, ответ. Логарифмы, напротив, работают немного по-другому. В логарифме вы знаете значение ответа и основания, но обычно не знаете значение показателя степени. В этом суть логарифма: найти значение показателя степени.

       

      Понимание логарифмической записи

      Наиболее важным аспектом понимания логарифмов, даже до того, как вы начнете использовать их для решения сложных математических задач, является понимание того, как преобразовать простую экспоненциальную форму в эквивалентный логарифм. Когда у вас есть основание b и вы применяете показатель степени x к is, вы получаете ответ a, как показано ниже.

                  b x = a

      Это же уравнение преобразуется в логарифм следующим образом:

                  log b (a) = x

      В логарифмическом представлении базовое значение выражается в виде нижнего индекса рядом с термином «log». Результат или ответ на основную задачу с показателем степени отображается в круглых скобках рядом с основанием, в то время как фактический показатель степени представляет собой решение логарифма. Используя действительные числа, задача работает следующим образом:

                  5 3 = 125 —> log 5 (125) = 3

      Как решать логарифмические уравнения

      Для решения основных задач с логарифмами вы можете использовать способность переводить туда и обратно между двумя форматами. Попробуйте найти n в следующем примере.

                  log 3 (729) = n

      Поскольку это соответствует 3 n = 729, для решения этой проблемы можно использовать любую из множества стратегий или инструментов. Вы можете ввести числа в калькулятор или использовать стратегию «угадай и проверь», подставляя маленькие числа, начиная с 2, пока не получишь результат 729.. В любом случае, вы можете вычислить, что n должно равняться 6.

      На уроке математики в старшей школе, таком как Алгебра 2 или предварительное исчисление, вы, вероятно, столкнетесь с такой проблемой на первых нескольких уроках логарифмов. Однако вопросы становятся более сложными. Уроки математики включают логарифмы из-за того, как часто эта концепция используется в математике более высокого уровня, физике, информатике, инженерии и других науках. Есть несколько концепций, с которыми вы регулярно сталкиваетесь и которые основаны на логарифмах, даже если вы не знали об этом. Одним из примеров является децибел, единица измерения мощности и амплитуды звука, которую мы ошибочно называем громкостью. Фактор рН различных химических веществ, таких как вода, также является логарифмом.

      Как только вы преодолеете основы, логарифмы станут более сложными, поскольку они представляют собой более сложные уравнения и решения. Используя логарифмы, вы можете точно вычислить отрицательные и дробные показатели, что сложнее сделать в простой экспоненциальной форме.

       

      Логарифмы в SAT/ACT

      В стандартных тестах, таких как ACT и SAT, вы должны помнить только основные правила логарифмирования. На самом деле, логарифмы очень редко встречаются в SAT и чаще появляются в математическом разделе ACT. В ACT базовые логарифмы появляются как минимум один раз за тестирование. Единственная более продвинутая концепция логарифма, которая появляется время от времени, — это логарифм произведения.

      Log B ( A * N ) = LOG B ( A ) + B () + B () + B (). 1 * b x 2 = b x 1+ 2 x правила экспоненты. В реальных числах это будет выглядеть так:

                  log 4 (16384) = log 4 (64) + log 4 (256)

      Это работает, потому что log 4 (64) = 3 и log 4 (256) = 4 и 64 * 26 = 16384.  В экспоненциальной форме вы можете найти ту же информацию.

                  4 3 * 4 4 = 4 3+4 = 4 7 = 16384

      . логарифмы, преподаваемые в начале процесса учителями математики в школе. Таким образом, он подпадает под заголовок основ концепции, из которых обычно авторы тестов ACT строят свои математические вопросы.

      Существует множество других более сложных задач на логарифм, с которыми вы столкнетесь на уроках математики в школе, однако это предел того, с чем вы столкнетесь на ACT. По крайней мере, это должно помочь вам начать изучение основ математики и подготовить вас к успеху в решении любых математических задач высокого уровня ACT в будущем.

      Решение экспоненциальных и логарифмических уравнений

      7.5 Решение экспоненциальных и логарифмических уравнений

      Цели обучения

      1. Решите показательные уравнения.
      2. Используйте формулу изменения основания для аппроксимации логарифмов.
      3. Решите логарифмические уравнения.

      Решение экспоненциальных уравнений

      Показательное уравнениеУравнение, которое включает переменную в качестве показателя степени. это уравнение, которое включает переменную в качестве одного из показателей. В этом разделе мы опишем два метода решения показательных уравнений. Во-первых, напомним, что экспоненциальные функции, определяемые формулой f(x)=bx, где b>0 и b≠1, являются взаимно однозначными; каждое значение в диапазоне соответствует ровно одному элементу домена. Следовательно, f(x)=f(y) подразумевает x=y. Обратное верно, потому что f — это функция. Это приводит к очень важному свойству экспоненциальных функций «один к одному». Учитывая, что b> 0 и b ≠ 1, мы имеем bx = by тогда и только тогда, когда x = y.:

      bx=by   если и только если        x=y

      Используйте это свойство для решения специальных экспоненциальных уравнений, в которых каждая сторона может быть записана в терминах одного и того же основания.

      Пример 1

      Решите: 32x−1=27.

      Решение:

      Начните с записи 27 как степени числа 3.

      32x−1=2732x−1=33

      Затем примените свойство однозначности экспоненциальных функций. Другими словами, установите показатели степени равными друг другу, а затем упростите.

      2x−1=32x=4x=2

      Ответ: 2

      Пример 2

      Решить: 161−3x=2.

      Решение:

      Начните с записи 16 как степени двойки, а затем примените правило степени для показателей степени.

      161−3x=2(24)1−3x=224(1−3x)=21

      Теперь, когда основания одинаковы, мы можем приравнять показатели степени друг к другу и упростить.

      4(1−3x)=14−12x=1−12x=−3x=−3−12=14

      Ответ: 14

      Попробуйте! Решите: 252x+3=125.

      Ответ: −34

      (нажмите, чтобы посмотреть видео)

      Во многих случаях мы не сможем приравнять основания. По этой причине мы развиваем второй метод решения показательных уравнений. Рассмотрим следующие уравнения:

      32=93?=1233=27

      Мы видим, что решение 3x=12 должно быть где-то между 2 и 3. Далее следует графическая интерпретация.

      Чтобы решить эту проблему, мы используем тот факт, что логарифмы являются взаимно однозначными функциями. Учитывая x, y> 0 взаимно однозначное свойство логарифмов. Учитывая b> 0 и b ≠ 1, где x, y> 0, мы имеем logb x=logb y тогда и только тогда, когда x=y. следует:

      logb x=logb y      if и только if      x=y

      Это свойство, как и свойства логарифма, позволяет нам решать экспоненциальные уравнения. Например, чтобы решить 3x=12, примените десятичный логарифм к обеим частям, а затем используйте свойства логарифма, чтобы изолировать переменную. 92,2618≈12✓

      Обратите внимание, что мы получаем , а не , умножая обе части на «логарифм»; мы применяем свойство логарифмических функций «один к одному», которое часто выражается как « берёт логарифм обеих сторон ». Общие шаги для решения показательных уравнений изложены в следующем примере.

      Пример 3

      Решите: 52x−1+2=9.

      Решение:

      • Шаг 1: Изолируйте экспоненциальное выражение.

        52x−1+2=952x−1=7

      • Шаг 2: Возьмем логарифм обеих частей. В этом случае мы возьмем десятичный логарифм обеих частей, чтобы можно было аппроксимировать наш результат на калькуляторе.

        log 52x−1=log 7

      • Шаг 3: Примените правило степени для логарифмов и затем решите.

        log 52x−1=log 7(2x−1)log 5=log 7  Distribute.2xlog 5−log 5=log 72xlog 5=log 5+log 7x=log 5+log 72log 5

      Это иррациональное число, которое можно приблизительно вычислить с помощью калькулятора. Позаботьтесь о том, чтобы сгруппировать числитель и произведение в знаменателе при вводе этого числа в калькулятор. Для этого используйте кнопки со скобками  (  и  )  :

      x=(log 5+log (7))/(2*log (5))≈1,1045

      Ответ: log 5+log 72log 5≈1,1045

      Пример 4

      Решите: e5x+3=1.

      Решение:

      Показательная функция уже изолирована, а основание равно e . Поэтому мы решили применить натуральный логарифм к обеим частям.

      e5x+3=1ln e5x+3=ln 1

      Примените правило степени для логарифмов, а затем упростите.

      ln e5x+3=ln 1(5x+3)ln e=ln 1  Вспомнить ln e=1 и ln1=0.(5x+3)⋅1=05x+3=0x=−35

      Ответ: − 35

      На большинстве калькуляторов есть только две кнопки логарифма, десятичный логарифм LOG и натуральный логарифм LN. Если мы хотим аппроксимировать log3 10, мы должны каким-то образом изменить это основание на 10 или 9.0088 е . Идея начинается с перезаписи логарифмической функции y=loga x в экспоненциальной форме.

      loga x=y  ⇒  x=ay

      Здесь x>0, поэтому мы можем применить свойство однозначности логарифмов. Примените основание логарифма b к обеим частям функции в экспоненциальной форме.

      x=aylogb x=logb ay

      И затем найти y .

      logb x=ylogb alogb xlogb a=y

      Замените y в исходную функцию, и мы получим очень важное изменение базовой формулыloga x=logb xlogb a; мы можем написать любое основание- a логарифмов по основанию- b логарифмов, используя эту формулу. :

      loga x=logb xlogb a

      Мы можем использовать это для аппроксимации log3 10 следующим образом.

      log3 10=log 10log 3≈2,0959    или    log3 10=ln 10ln 3≈2,0959

      Обратите внимание, что результат не зависит от выбора основания. Другими словами, мы можем приблизить логарифм любого заданного основания на калькуляторе, разделив логарифм аргумента на логарифм этого заданного основания.

      Пример 5

      Приблизительно log7 120 с точностью до сотой.

      Решение:

      Примените изменение базовой формулы и используйте калькулятор.

      log7 120=log 120log 7

      На калькуляторе

      log (120)/log (7)≈2,46

      Ответ: 2,46

      23 23 Решите: 23x+1−4=1. Дайте точный и приблизительный ответ с округлением до четвертого знака после запятой.

      Ответ: log 5−log 23log 2≈0,4406

      (нажмите, чтобы посмотреть видео)

      Решение логарифмических уравнений

      Логарифмическое уравнениеУравнение, содержащее логарифм с переменным аргументом. это уравнение, которое включает логарифм с переменным аргументом. Некоторые логарифмические уравнения можно решить, используя свойство логарифмов «один к одному». Это верно, когда один логарифм с одинаковым основанием может быть получен с обеих сторон от знака равенства.

      Пример 6

      Решите: log2 (2x−5)−log2 (x−2)=0.

      Решение:

      Мы можем получить два равных логарифма по основанию 2, добавив log2 (x−2) к обеим частям уравнения.

      log2 (2x−5)−log2 (x−2)=0log2 (2x−5)=log2 (x−2)

      Здесь основания одинаковы, поэтому мы можем применить свойство однозначности и установить аргументы равными друг другу.

      log2 (2x−5)=log2 (x−2)2x−5=x−2x=3

      Проверка x=3 в исходном уравнении:

      log2 (2(3)−5)=log2 (( 3)−2)log2 1=log2 10=0     ✓

      Ответ: 3

      При решении логарифмических уравнений проверка очень важна, так как могут быть получены посторонние решения. Свойства логарифма применяются только для значений в области данного логарифма. А при работе с переменными аргументами, такими как log (x−2), значение x неизвестно до конца этого процесса. Логарифмическое выражение log (x−2) определено только для значений x>2.

      Пример 7

      Решите: log (3x−4)=log (x−2).

      Решение:

      Примените свойство один к одному логарифмов (установите аргументы равными друг другу), а затем найдите x .

      log (3x−4)=log (x−2)3x−4=x−22x=2x=1

      При проверке встречается логарифм отрицательного числа:

      log (x−2)= log (1−2)=log (−1)  не определено

      Попробуйте это на калькуляторе, что он говорит? Здесь x=1 не находится в области log (x−2). Поэтому наше единственное возможное решение является посторонним, и мы заключаем, что у этого уравнения нет решений.

      Ответ: Нет решения, Ø.

      Внимание: Решение логарифмических уравнений иногда приводит к посторонним решениям — мы должны проверить наши ответы.

      Попробуйте! Решите: ln (x2−15)−ln (2x)=0.

      Ответ: 5

      (нажмите, чтобы посмотреть видео)

      Во многих случаях мы не сможем получить два равных логарифма. Для решения таких уравнений воспользуемся определением логарифма. Если b>0, где b≠1, то logb x=y подразумевает, что by=x. Рассмотрим следующие десятичные десятичные уравнения:

      Мы видим, что решение log x=0,5 будет где-то между 1 и 10. Далее следует графическая интерпретация.

      Найти 90,5≈3,1623

      Ответ от 1 до 10 — это то, что мы ожидали. Проверьте это на калькуляторе.

      log 3,1623≈5✓

      Пример 8

      Решите: log3 (2x−5)=2.

      Решение:

      Применить определение логарифма.

      log3 (2x−5)=2  ⇒  2x−5=32

      Решите полученное уравнение.

      2x−5=92x=14x=7

      Проверить.

      log3 (2(7)−5)=?2       log3 (9)=2   ✓

      Ответ: 7

      Чтобы применить определение, нам потребуется переписать логарифмические выражения в виде одиночного логарифма с коэффициентом 1.1.1. Общие шаги решения логарифмических уравнений показаны в следующем примере.

      Пример 9

      Решите: log2 (x−2)+log2 (x−3)=1.

      Решение:

      • Шаг 1: Запишите все логарифмические выражения в виде одного логарифма с коэффициентом 1. В этом случае примените правило произведения для логарифмов.

        log2 (x−2)+log2 (x−3)=1log2 [(x−2)(x−3)]=1

      • Шаг 2: Используйте определение и перепишите логарифм в экспоненциальной форме.

        log2 [(x−2)(x−3)]=1  ⇒  (x−2)(x−3)=21

      • Шаг 3: Решите полученное уравнение. Здесь мы можем решить факторингом.

        (x−2)(x−3)=2×2−5x+6=2×2−5x+4=0(x−4)(x−1)=0x−4=0    или    x−1=0x=4x= 1

      • Шаг 4: Проверка. Этот шаг обязателен.

      В этом примере x=1 не входит в область определения данного логарифмического выражения и является лишним. Единственное решение х=4.

      Ответ: 4

      Пример 10

      Решите: log (x+15)−1=log (x+6).

      Решение:

      Начните с записи всех логарифмических выражений с одной стороны и констант с другой.

      log (x+15)−1=log (x+6)log (x+15)−log (x+6)=1

      Применить правило частных для логарифмов как средство получения единичного логарифма с коэффициентом 1.

      log (x+15)−log (x+6)=1log (x+15x+6)=1

      Это десятичный логарифм; поэтому используйте 10 в качестве основы при применении определения.

      х+15х+6=101х+15=10(х+6)х+15=10х+60-9х=45х=-5

      Проверить.

      log (x+15)−1=log (x+6)log (−5+15)−1=log (−5+6)log 10−1=log 11−1=00=0    ✓

      Ответ: −5

      Попробуйте! Решите: log2 (x)+log2 (x−1)=1.

      Ответ: 2

      (нажмите, чтобы посмотреть видео)

      Пример 11

      Найдите обратное: f(x)=log2 (3x−4).

      Решение:

      Начните с замены обозначения функции f(x) на y .

      f(x)=log2 (3x−4)y=log2 (3x−4)

      Поменять местами x и y , а затем найти y .

      x=log2 (3y−4)  ⇒  3y−4=2x3y=2x+4y=2x+43

      Полученная функция является обратной f . Представьте ответ, используя функциональную запись.

      Ответ: f−1(x)=2x+43

      Основные выводы

      • Если каждую часть экспоненциального уравнения можно выразить с помощью одного и того же основания, приравняйте показатели и решите.
      • Чтобы решить общее показательное уравнение, сначала выделите показательное выражение, а затем примените соответствующий логарифм к обеим частям. Это позволяет нам использовать свойства логарифмов для решения переменной.
      • Изменение базовой формулы позволяет нам использовать калькулятор для вычисления логарифмов. Логарифм числа равен десятичному логарифму числа, деленному на десятичный логарифм по данному основанию.
      • Если в каждой части уравнения можно выделить один логарифм с одинаковым основанием, то приравняйте аргументы и решите.
      • Чтобы решить общее логарифмическое уравнение, сначала выделите логарифм с коэффициентом 1, а затем примените определение. Решите полученное уравнение.
      • Действия по решению логарифмических уравнений иногда приводят к посторонним решениям. Поэтому проверка обязательна.

      Упражнения по теме

        Часть A. Решение экспоненциальных уравнений

          Решите, используя свойство однозначности экспоненциальных функций.

        1. 3x=81

        2. 2−x=16

        3. 5x−1=25

        4. 3x+4=27

        5. 25x−2=16

        6. 23x+7=8

        7. 812x+1=3

        8. 643x−2=2

        9. 92-3x-27=0

        10. 81-5x-32=0

        11. 16×2−2=0

        12. 4×2−1−64=0

        13. 9х(х+1)=81

        14. 4x(2x+5)=64

        15. 100×2-107x-3=0

        16. e3(3×2−1)−e=0

          Решить. Дайте точный ответ и примерный ответ, округлив его до тысячных.

        1. 3x=5

        2. 7х=2

        3. 4x=9

        4. 2x=10

        5. 5x−3=13

        6. 3x+5=17

        7. 72x+5=2

        8. 35x−9=11

        9. 54x+3+6=4

        10. 107x−1−2=1

        11. е2х-3-5=0

        12. е5х+1-10=0

        13. 63x+1−3=7

        14. 8−109x+2=9

        15. 15−e3x=2

        16. 7+е4х+1=10

        17. 7−9e−x=4

        18. 3−6e−x=0

        19. 5×2=2

        20. 32×2-x=1

        21. 100e27x=50

        22. 6e12x=2

        23. 31+е-х=1

        24. 21+3е-х=1

          Найдите точки пересечения x и y заданной функции.

        1. f(x)=3x+1−4

        2. f(x)=23x−1−1

        3. f(x)=10x+1+2

        4. f(x)=104x−5

        5. f(x)=ex−2+1

        6. f(x)=ex+4−4

          Используйте замену u для решения следующей задачи.

        1. 32x−3x−6=0 (Подсказка: пусть u=3x)

        2. 22х+2х-20=0

        3. 102х+10х-12=0

        4. 102x−10x−30=0

        5. е2х-3ех+2=0

        6. е2х-8ех+15=0

          Используйте формулу изменения основания, чтобы приблизить следующее число с точностью до сотых.

        1. лог2 5

        2. log3 7

        3. лог5 (23)

        4. лог7 (15)

        5. лог1/2 10

        6. лог2/3 30

        7. лог2 5

        8. log2 63

        9. Если не остановить новый штамм вируса гриппа, он может очень быстро распространиться от одного человека к другому. Количество затронутых людей можно смоделировать по формуле P(t)=e0,22t, где t представляет количество дней, в течение которых вирус может беспрепятственно распространяться. Оцените, сколько дней потребуется, чтобы заразиться 1000 человек.

        10. Население небольшого городка растет в соответствии с функцией P(t)=12 500(1,02)t, где t представляет собой время в годах с момента последней переписи. Используйте функцию, чтобы определить, сколько лет потребуется, чтобы население увеличилось до 25 000 человек.

        Часть B: Решение логарифмических уравнений

          Решите, используя свойство однозначности логарифмов.

        1. log5 (2x+4)=log5 (3x−6)

        2. log4 (7x)=log4 (5x+14)

        3. log2 (x−2)−log2 (6x−5)=0

        4. пер (2x−1)=перн (3x)

        5. log (x+5)−log (2x+7)=0

        6. лн (х2+4х)=2лн (х+1)

        7. log3 2+2log3 x=log3 (7x−3)

        8. 2log x-log 36=0

        9. пер (х+3)+лн (х+1)=лн 8

        10. log5 (x−2)+log5 (x−5)=log5 10

          Решить.

        1. log2 (3x−7)=5

        2. log3 (2x+1)=2

        3. log (2x+20)=1

        4. log4 (3x+5)=12

        5. log3 x2=2

        6. log (x2+3x+10)=1

        7. ln (x2−1)=0

        8. log5 (x2+20)−2=0

        9. log2 (x−5)+log2 (x−9)=5

        10. log2 (x+5)+log2 (x+1)=5

        11. log4 x+log4 (x−6)=2

        12. log6 x+log6 (2x−1)=2

        13. log3 (2x+5)−log3 (x−1)=2

        14. log2 (x+1)−log2 (x−2)=4

        15. ln x−ln (x−1)=1

        16. ln (2x+1)−ln x=2

        17. 2log3 x=2+log3 (2x−9)

        18. 2log2 x=3+log2 (x−2)

        19. log2 (x−2)=2−log2 x

        20. log2 (x+3)+log2 (x+1)−1=0

        21. лог x−log (x+1)=1

        22. log2 (x+2)+log2 (1−x)=1+log2 (x+1)

          Найдите точки пересечения x и y заданной функции.

        1. f(x)=log (x+3)−1

        2. f(x)=log (x−2)+1

        3. f(x)=log2 (3x)−4

        4. f(x)=log3 (x+4)−3

        5. f(x)=ln (2x+5)−6

        6. f(x)=ln (x+1)+2

          Найдите обратную следующие функции.

        1. f(x)=log2 (x+5)

        2. f(x)=4+log3 x

        3. f(x)=log (x+2)−3

        4. f(x)=ln (x−4)+1

        5. f(x)=ln (9x−2)+5

        6. f(x)=log6 (2x+7)−1

        7. г(х)=e3x

        8. г(х)=10-2х

        9. г(х)=2х+3

        10. г(х)=32х+5

        11. г(х)=10х+4-3

        12. г(х)=e2x−1+1

          Решить.

        1. log (9x+5)=1+log (x−5)

        2. 2+log2 (x2+1)=log2 13

        3. е5х-2-е3х=0

        4. 3×2−11=70

        5. 23x−5=0

        6. log7 (x+1)+log7 (x−1)=1

        7. ln (4x−1)−1=ln x

        8. log (20x+1)=log x+2

        9. 31+е2х=2

        10. 2е-3х=4

        11. 2е3х=е4х+1

        12. 2log x+log x−1=0

        13. 3log x=log (x−2)+2log x

        14. 2ln 3+ln x2=ln (x2+1)

        15. В химии pH является мерой кислотности и определяется по формуле pH=−log (H+), где H+ – концентрация ионов водорода (измеряется в молях водорода на литр раствора). Определите концентрацию ионов водорода, если рН раствора 4.

        16. Громкость звука, L в децибелах (дБ), определяется по формуле L=10log (I/10−12), где I представляет интенсивность звука в ваттах на квадратный метр. Определите интенсивность сигнала тревоги, который издает звук мощностью 120 дБ.

        Часть C: Дискуссионная доска

      1. Исследовать и обсудить историю и использование логарифмической линейки.

      2. Исследуйте и обсуждайте реальные приложения, использующие логарифмы.

      Ответы

      1. 4

      2. 3

      3. 65

      4. −38

      5. 16

      6. ±12

      7. −2, 1

      8. 12, 3

      9. лог 5лог 3≈1,465

      10. log 3log 2≈1,585

      11. 3log 5+log 13log 5≈4,594

      12. log 2−5log 72log 7≈−2,322

      13. Ø

      14. 3+ln 52≈2,305

      15. 1−log 63log 6≈0,095

      16. лн 133≈0,855

      17. лн 3≈1,099

      18. ±log 2log 5≈±0,656

      19. −ln 227≈−0,026

      20. −ln 2≈−0,693

      21. x — перехват: (2log 2−log 3log 3,0); г -пересечение: (0,−1)

      22. x — перехват: нет; г -перехват: (0,12)

      23. x — перехват: нет; г -перехват: (0,1+e2e2)

      24. 1

      25. журнал 3

      26. 0, пер 2

      27. 2,32

      28. −0,25

      29. −3,32

      30. 1,16

      31. Приблизительно 31 день

      1. 10

      2. 35

      3. −2

      4. 12, 3

      5. 1

      6. 13

      7. −5

      8. ±3

      9. ±2

      10. 13

      11. 8

      12. 2

      13. ее-1

      14. 9

      15. 1+5

      16. Ø

      17. x -пересечение: (7,0); г -перехват: (0, log 3−1)

      18. x — точка пересечения: (163, 0); г -перехват: Нет

      19. x -пересечение: (e6−52,0); г -перехват: (0,ln 5−6)

      20. f−1(x)=2x−5

      21. f−1(x)=10x+3−2

      22. f−1(x)=ex−5+29

      23. г−1(x)=ln x3

      24. г-1(х)=log2 х-3

      25. г−1(x)=log (x+3)−4

      26. 55

      27. 1

      28. лог2 53

      29. 14−е

      30. л (1/2)2

      31. лн 2−1

      32. Ø

      33. 10-4 моль на литр

      1. Ответ может отличаться

      10.

      5 Решение экспоненциальных и логарифмических уравнений — Алгебра среднего уровня 2e

      Цели обучения

      К концу этого раздела вы сможете:

      • Решение логарифмических уравнений с использованием свойств логарифмов
      • Решение показательных уравнений с использованием логарифмов
      • Использование экспоненциальных моделей в приложениях

      Приготовься 10.13

      Прежде чем приступить к работе, пройдите этот тест на готовность.

      Решите: x2=16.x2=16.
      Если вы пропустили эту проблему, просмотрите пример 6.46.

      Приготовься 10.14

      Решите: x2−5x+6=0.x2−5x+6=0.
      Если вы пропустили эту проблему, просмотрите пример 6.45.

      Приготовься 10.15

      Решите: x(x+6)=2x+5.x(x+6)=2x+5.
      Если вы пропустили эту проблему, просмотрите пример 6.47.

      Решение логарифмических уравнений с использованием свойств логарифмов

      В разделе, посвященном логарифмическим функциям, мы решили некоторые уравнения, переписав уравнение в экспоненциальной форме. Теперь, когда у нас есть свойства логарифмов, у нас есть дополнительные методы, которые мы можем использовать для решения логарифмических уравнений.

      Если наше уравнение имеет два логарифма, мы можем использовать свойство, которое говорит, что если logaM=logaNlogaM=logaN, то верно, что M=N.M=N. Это однозначное свойство логарифмических уравнений.

      Однозначное свойство логарифмических уравнений

      Для M>0,N>0,a>0,M>0,N>0,a>0 и a≠1a≠1 — любое действительное число:

      IflogaM=logaN, thenM=N.IflogaM= logaN, тогда M=N.

      Чтобы использовать это свойство, мы должны быть уверены, что обе части уравнения записаны с одним и тем же основанием.

      Помните, что логарифмы определены только для положительных вещественных чисел. Проверьте свои результаты в исходном уравнении. Возможно, вы получили результат, который дает логарифм нуля или отрицательного числа.

      Пример 10.38

      Решите: 2log5x=log581.2log5x=log581.

      Решение

      2log5x=log5812log5x=log581
      Используйте свойство Power. log5x2=log581log5x2=log581
      Использовать свойство «один к одному», если logaM=logaNlogaM=logaN, то M=NM=N х2=81х2=81.
      Решение с использованием свойства квадратного корня. х=±9х=±9
      Мы исключаем x=−9x=−9, поскольку мы не можем логарифмировать отрицательное число. х=9,х=-9х=9,х=-9
      Чек.
      x=92log5x=log5812log59=?log581log592=?log581log581=log581✓x=92log5x=log5812log59=?log581log592=?log581log581=log581✓

      Попытайся 10,75

      Решить: 2log3x=log3362log3x=log336

      Попытайся 10,76

      Решить: 3logx=log643logx=log64

      Еще одна стратегия, используемая для решения логарифмических уравнений, состоит в том, чтобы сжать суммы или разности в единый логарифм.

      Пример 10.39

      Решите: log3x+log3(x−8)=2.log3x+log3(x−8)=2.

      Решение

      log3x+log3(x-8)=2log3x+log3(x-8)=2
      Используйте свойство продукта, logaM+logaN=logaM⋅NlogaM+logaN=logaM⋅N. log3x(x−8)=2log3x(x−8)=2
      Переписать в экспоненциальной форме. 32=х(х-8)32=х(х-8)
      Упрощение. 9=х2-8х9=х2-8х
      Вычтите 9 с каждой стороны. 0=х2-8х-90=х2-8х-9
      Фактор. 0=(х-9)(х+1)0=(х-9)(х+1)
      Используйте свойство Zero-Product. х-9=0,х+1=0х-9=0,х+1=0
      Решите каждое уравнение. х=9,х=-1х=9,х=-1
      Чек.
      x=-1log3x+log3(x-8)=2log3(-1)+log3(-1-8)=?2x=-1log3x+log3(x-8)=2log3(-1)+log3 (−1−8)=?2
      Мы не можем взять журнал отрицательного числа.
      x=9log3x+log3(x-8)=2log39+log3(9-8)=?22+0=?22=2✓x=9log3x+log3(x-8)=2log39+log3(9-8) =?22+0=?22=2✓

      Попытайся 10,77

      Решить: log2x+log2(x−2)=3log2x+log2(x−2)=3

      Попытайся 10,78

      Решить: log2x+log2(x−6)=4log2x+log2(x−6)=4

      Когда с обеих сторон есть логарифмы, мы сжимаем каждую сторону в один логарифм. Не забудьте использовать свойство Power по мере необходимости.

      Пример 10.40

      Решите: log4(x+6)−log4(2x+5)=−log4x.log4(x+6)−log4(2x+5)=−log4x.

      Решение

      log4(x+6)−log4(2x+5)=−log4xlog4(x+6)−log4(2x+5)=−log4x
      Используйте свойство Quotient слева и свойство Power справа. log4(x+62x+5)=log4x-1log4(x+62x+5)=log4x-1
      Переписать x−1=1xx−1=1x. log4(x+62x+5)=log41xlog4(x+62x+5)=log41x
      Используйте свойство «один к одному», если logaM=logaNlogaM=logaN, то M=NM=N. х+62х+5=1хх+62х+5=1х
      Решите рациональное уравнение. х(х+6)=2х+5х(х+6)=2х+5
      Распределить. х2+6х=2х+5х2+6х=2х+5
      Пишите в стандартной форме. х2+4х-5=0х2+4х-5=0
      Фактор. (х+5)(х-1)=0(х+5)(х-1)=0
      Используйте свойство Zero-Product. х+5=0,х-1=0х+5=0,х-1=0
      Решите каждое уравнение. х=-5,х=1х=-5,х=1
      Чек.
      Мы оставляем вам чек.

      Попытайся 10,79

      Решите: log(x+2)−log(4x+3)=−logx. log(x+2)−log(4x+3)=−logx.

      Попытайся 10.80

      Решите: log(x−2)−log(4x+16)=log1x.log(x−2)−log(4x+16)=log1x.

      Решение экспоненциальных уравнений с использованием логарифмов

      В разделе, посвященном экспоненциальным функциям, мы решили некоторые уравнения, записав обе части уравнения с одним и тем же основанием. Затем мы написали новое уравнение, приравняв показатели степени.

      Не всегда возможно или удобно писать выражения с одной основой. В этом случае мы часто берем десятичный логарифм или натуральный логарифм обеих частей после выделения экспоненты.

      Пример 10.41

      Решите 5x=11,5x=11. Найдите точный ответ и аппроксимируйте его до трех знаков после запятой.

      Решение
      5х=115х=11
      Поскольку экспонента изолирована, возьмем логарифм обеих частей.
      Используйте свойство Power, чтобы получить xx как множитель, а не показатель степени.

      Решите для х.х. Найдите точный ответ.

      Приблизительный ответ.

      log5x=log11xlog5=log11x=log11log5x≈1,490log5x=log11xlog5=log11x=log11log5x≈1,490
      Поскольку 51=551=5 и 52=25,52=25, имеет ли смысл 51,490≈11?51,490≈11?

      Попытайся 10,81

      Решите 7x=43,7x=43. Найдите точный ответ и аппроксимируйте его до трех знаков после запятой.

      Попытайся 10,82

      Решите 8x=98,8x=98. Найдите точный ответ и аппроксимируйте его до трех знаков после запятой.

      Когда мы логарифмируем обе части, мы получаем один и тот же результат независимо от того, используем ли мы десятичный или натуральный логарифм (попробуйте использовать натуральный логарифм в последнем примере. Вы получили тот же результат?) Когда экспонента имеет основание e , мы используем натуральный логарифм.

      Пример 10.42

      Решите 3ex+2=24,3ex+2=24. Найдите точный ответ и аппроксимируйте его до трех знаков после запятой.

      Решение

      3ex+2=243ex+2=24
      Изолируйте экспоненту, разделив обе части на 3. ех+2=8ех+2=8
      Возьмем натуральный логарифм обеих частей. lnex+2=ln8lnex+2=ln8
      Используйте свойство Power, чтобы получить xx как множитель, а не показатель степени. (x+2)lne=ln8(x+2)lne=ln8
      Используйте свойство lne=1lne=1 для упрощения. х+2=ln8x+2=ln8
      Решите уравнение. Найдите точный ответ. х=ln8-2x=ln8-2
      Приблизительный ответ. х ≈ 0,079 х ≈ 0,079

      Попытайся 10,83

      Решите 2ex−2=18,2ex−2=18. Найдите точный ответ и аппроксимируйте его до трех знаков после запятой.

      Попытайся 10,84

      Решите 5e2x=25,5e2x=25. Найдите точный ответ и аппроксимируйте его до трех знаков после запятой.

      Использование экспоненциальных моделей в приложениях

      В предыдущих разделах мы смогли решить некоторые приложения, которые были смоделированы с помощью показательных уравнений. Теперь, когда у нас есть так много вариантов решения этих уравнений, мы можем решать больше приложений.

      Мы снова воспользуемся формулами сложных процентов и приведем их здесь для справки.

      Сложные проценты

      Для основного долга, P , инвестированного под процентную ставку, r , на t лет, новый баланс, A , составляет:

      A=P(1+rn)nt при начислении процентов n раз в год. A=Pert при непрерывном начислении. A=P(1+rn)nt при начислении n раз в год. A=Pert при непрерывном начислении.

      Пример 10.43

      Родители Джермаэля вложили 10 000 долларов на его расходы в колледже в день его первого дня рождения. Они надеются, что инвестиции будут стоить 50 000 долларов, когда ему исполнится 18 лет. Если проценты будут накапливаться непрерывно, какой примерно темп роста им потребуется для достижения своей цели?

      Решение

      А=50 000 долларов А=50 000 долларов
      Р = 10 000 долл. США Р = 10 000 долл. США
      Определите переменные в формуле г=?г=?
      t=17лет=17лет
      А=ПертА=Перт
      Подставьте значения в формулу. 50 000=10 000эр·1750 000=10 000эр·17
      Решите для r.r. Разделите каждую сторону на 10000. 5=e17r5=e17r
      Возьмите натуральное бревно с каждой стороны. ln5=lne17rln5=lne17r
      Используйте свойство Power. ln5=17rlneln5=17rlne
      Упрощение. пер5=17рлн5=17р
      Разделите каждую сторону на 17. лн517=рлн517=р
      Приблизительный ответ. r≈0,095r≈0,095
      Преобразование в проценты. г≈9,5%г≈9,5%
      Им нужно, чтобы скорость роста составляла примерно 9,5%9,5%.

      Попытайся 10,85

      Гектор инвестирует 10 000 долларов США 10 000 долларов США в возрасте 21 года. Он надеется, что инвестиции будут стоить 150 000 долларов США 150 000 долларов США, когда ему исполнится 50 лет. Если проценты постоянно увеличиваются, какой примерно темп роста ему потребуется для достижения своей цели?

      Попытайся 10,86

      Рэйчел инвестирует 15 000 долларов США 15 000 долларов США в возрасте 25 лет. Она надеется, что инвестиции будут стоить 90 000 долларов США 90 000 долларов США, когда ей исполнится 40 лет. Если проценты постоянно увеличиваются, какой примерно темп роста ей потребуется для достижения своей цели?

      Мы видели, что рост и затухание моделируются экспоненциальными функциями. Для роста и распада мы используем формулу A=A0ekt.A=A0ekt. Экспоненциальный рост имеет положительную скорость роста или константу роста, kk, а экспоненциальный спад имеет отрицательную скорость роста или константу спада, к .

      Экспоненциальный рост и упадок

      Для исходной суммы A0,A0, которая растет или уменьшается со скоростью k , в течение определенного времени t , окончательная сумма A составляет:

      A=A0ektA=A0ekt

      Теперь мы можем решать приложения, которые дают нам достаточно информации для определения скорости роста. Затем мы можем использовать эту скорость роста для прогнозирования других ситуаций.

      Пример 10.44

      Исследователи зафиксировали, что популяция определенных бактерий выросла со 100 до 300 за 3 часа. При такой скорости роста сколько бактерий будет через 24 часа после начала эксперимента?

      Решение

      Эта проблема требует двух основных шагов. Сначала мы должны найти неизвестную скорость, к . Затем мы используем это значение k , чтобы найти неизвестное количество бактерий.

      Определите переменные в формуле. A=300A0=100k=?t=3hoursA=A0ektA=300A0=100k=?t=3hoursA=A0ekt
      Подставьте значения в формулу. 300=100эк·3300=100эк·3
      Решите для kk. Разделите каждую сторону на 100. 3=e3k3=e3k
      Возьмите натуральное бревно с каждой стороны. ln3=lne3kln3=lne3k
      Используйте свойство Power. ln3=3klneln3=3klne
      Упрощение. лн3=3клн3=3к
      Разделите каждую сторону на 3. лн33=клн33=к
      Приблизительный ответ. к≈0,366к≈0,366
      Мы используем эту скорость роста, чтобы предсказать количество бактерий через 24 часа. A=?A0=100k=ln33t=24 часаA=A0ektA=?A0=100k=ln33t=24hoursA=A0ekt
      Подставьте значения. А=100eln33·24A=100eln33·24
      Оценить. А≈656 100 А≈656 100
      При такой скорости роста можно ожидать 656 100 бактерий.

      Попытайся 10,87

      Исследователи зафиксировали, что популяция определенных бактерий выросла со 100 до 500 за 6 часов. При такой скорости роста сколько бактерий будет через 24 часа после начала эксперимента?

      Попытайся 10,88

      Исследователи зафиксировали, что популяция определенных бактерий уменьшилась с 700 000 до 400 000 через 5 часов после введения лекарства. При такой скорости разложения сколько бактерий останется через 24 часа после начала эксперимента?

      Радиоактивные вещества распадаются или разлагаются по формуле экспоненциального распада. Время, за которое вещество распадается до половины своего исходного количества, называется периодом полураспада вещества.

      Как и в предыдущем примере, мы можем использовать данную информацию для определения константы распада, а затем использовать эту константу для ответа на другие вопросы.

      Пример 10.45

      Период полураспада радия-226 составляет 1590 лет. Какая часть образца массой 100 мг останется через 500 лет?

      Решение

      Эта проблема требует двух основных шагов. Сначала мы должны найти постоянную распада k . Если мы начнем со 100 мг, в период полувыведения останется 50 мг. Мы будем использовать эту информацию, чтобы найти к . Затем мы используем это значение k , чтобы найти количество выборки, которое останется через 500 лет.

      Определите переменные в формуле.
      A=50A0=100k=?t=1590летA=A0эктA=50A0=100k=?t=1590летA=A0экт
      Подставьте значения в формулу. 50=100эк·159050=100эк·1590
      Найти kk. Разделите каждую сторону на 100. 0,5=e1590k0,5=e1590k
      Возьмите натуральное бревно с каждой стороны. ln0.5=lne1590kln0.5=lne1590k
      Используйте свойство Power. ln0.5=1590klneln0.5=1590klne
      Упрощение. ln0.5=1590kln0.5=1590k
      Разделите каждую сторону на 1590. ln0.51590=точный ответln0.51590=точный ответ
      Мы используем эту скорость роста, чтобы предсказать количество, которое останется через 500 лет. А=?A0=100k=ln0,51590t=500летA=A0эктA=?A0=100k=ln0.51590t=500летA=A0экт
      Подставьте значения. А=100eln0,51590·500A=100eln0,51590·500
      Оценить. А≈80,4 мгА≈80,4 мг
      Через 500 лет останется примерно 80,4 мг.

      Попытайся 10,89

      Период полураспада магния-27 составляет 9,45 минут. Какой объем образца массой 10 мг останется через 6 минут?

      Попытайся 10,90

      Период полураспада радиоактивного йода составляет 60 дней. Какой объем образца массой 50 мг останется через 40 дней?

      Раздел 10.5 Упражнения

      Практика ведет к совершенству

      Решение логарифмических уравнений с использованием свойств логарифмов

      В следующих упражнениях найдите x .

      288.

      log464=2log4xlog464=2log4x

      289.

      лог49=2логхлог49=2logx

      290.

      3log3x=log3273log3x=log327

      291.

      3log6x=log6643log6x=log664

      292.

      log5(4x-2)=log510log5(4x-2)=log510

      293.

      log3(x2+3)=log34xlog3(x2+3)=log34x

      294.

      log3x+log3x=2log3x+log3x=2

      295.

      log4x+log4x=3log4x+log4x=3

      296.

      log2x+log2(x−3)=2log2x+log2(x−3)=2

      297.

      log3x+log3(x+6)=3log3x+log3(x+6)=3

      298.

      logx+log(x+3)=1logx+log(x+3)=1

      299.

      logx+log(x-15)=2logx+log(x-15)=2

      300.

      log(x+4)-log(5x+12)=-logxlog(x+4)-log(5x+12)=-logx

      301.

      log(x-1)-log(x+3)=log1xlog(x-1)-log(x+3)=log1x

      302.

      log5(x+3)+log5(x−6)=log510log5(x+3)+log5(x−6)=log510

      303.

      log5(x+1)+log5(x−5)=log57log5(x+1)+log5(x−5)=log57

      304.

      log3(2x-1)=log3(x+3)+log33log3(2x-1)=log3(x+3)+log33

      305.

      лог(5х+1)=лог(х+3)+лог2лог(5х+1)=лог(х+3)+лог2

      Решение экспоненциальных уравнений с использованием логарифмов

      В следующих упражнениях решите каждое показательное уравнение. Найдите точный ответ и аппроксимируйте его до трех знаков после запятой.

      306.

      3x=893x=89

      307.

      2x=742x=74

      308.

      5x=1105x=110

      309.

      4x=1124x=112

      310.

      ех=16ех=16

      311.

      ех=8ех=8

      312.

      (12)х=6(12)х=6

      313.

      (13)х=8(13)х=8

      314.

      4ex+1=164ex+1=16

      315.

      3ex+2=93ex+2=9

      316.

      6e2x=246e2x=24

      317.

      2e3x=322e3x=32

      318.

      14ex=314ex=3

      319.

      13ex=213ex=2

      320.

      ех+1+2=16ех+1+2=16

      321.

      ex−1+4=12ex−1+4=12

      В следующих упражнениях решите каждое уравнение.

      322.

      33x+1=8133x+1=81

      323.

      64x−17=21664x−17=216

      324.

      ex2e14=e5xex2e14=e5x

      325.

      ex2ex=e20ex2ex=e20

      326.

      loga64=2loga64=2

      327.

      loga81=4loga81=4

      328.

      lnx=-8lnx=-8

      329.

      lnx=9lnx=9

      330.

      log5(3x−8)=2log5(3x−8)=2

      331.

      log4(7x+15)=3log4(7x+15)=3

      332.

      lne5x=30lne5x=30

      333.

      lne6x=18lne6x=18

      334.

      3logx=log1253logx=log125

      335.

      7log3x=log31287log3x=log3128

      336.

      log6x+log6(x-5)=log624log6x+log6(x-5)=log624

      337.

      log9x+log9(x−4)=log912log9x+log9(x−4)=log912

      338.

      log2(x+2)-log2(2x+9)=-log2xlog2(x+2)-log2(2x+9)=-log2x

      339.

      log6(x+1)-log6(4x+10)=log61xlog6(x+1)-log6(4x+10)=log61x

      В следующих упражнениях найдите x , что даст точный ответ, а также приближение к трем знакам после запятой.

      340.

      6х=916х=91

      341.

      (12)х=10(12)х=10

      342.

      7ex−3=357ex−3=35

      343.

      8ex+5=568ex+5=56

      Использование экспоненциальных моделей в приложениях

      В следующих упражнениях решите.

      344.

      Сунг Ли инвестирует 5 000 долларов 5 000 долларов в возрасте 18 лет. Он надеется, что инвестиции будут стоить 10 000 долларов 10 000 долларов, когда ему исполнится 25 лет. Если проценты постоянно увеличиваются, какой примерно темп роста ему потребуется для достижения своей цели? Это разумное ожидание?

      345.

      Алиса инвестирует 15 000 долларов 15 000 долларов в возрасте 30 лет из подписного бонуса на новой работе. Она надеется, что инвестиции будут стоить 30 000 долларов 30 000 долларов, когда ей исполнится 40 лет. Если проценты будут увеличиваться постоянно, какой примерно темп роста ей потребуется для достижения своей цели?

      346.

      Coralee вкладывает 5000 долларов 5000 долларов на счет, который ежемесячно начисляет проценты и зарабатывает 7%.7%. Через какое время ее деньги удвоятся?

      347.

      Симона инвестирует 8000 долларов 8000 долларов на счет, который ежеквартально начисляет проценты и зарабатывает 5%. 5%. Через какое время его деньги удвоятся?

      348.

      Исследователи зафиксировали, что популяция определенных бактерий сократилась со 100 000 до 100 за 24 часа. При такой скорости разложения сколько бактерий останется за 16 часов?

      349.

      Исследователи зафиксировали, что популяция определенных бактерий уменьшилась с 800 000 до 500 000 через 6 часов после введения лекарства. При такой скорости разложения сколько бактерий будет через 24 часа?

      350.

      Вирусу требуется 6 дней, чтобы удвоить свою первоначальную популяцию (A=2A0).(A=2A0). За какое время его население утроится?

      351.

      Бактерии удваивают свою первоначальную популяцию за 24 часа (A=2A0).(A=2A0). Насколько большим будет его население через 72 часа?

      352.

      Углерод-14 используется для археологического радиоуглеродного датирования. Его период полураспада составляет 5730 лет. Сколько 100-граммового образца углерода-14 останется через 1000 лет?

      353.

      Радиоактивный технеций-99m часто используется в диагностической медицине, так как он имеет относительно короткий период полураспада, но действует достаточно долго, чтобы провести необходимые анализы пациента. Если период его полураспада составляет 6 часов, сколько радиоактивного материала из 0,5 мл инъекции будет в организме через 24 часа?

      Письменные упражнения

      354.

      Объясните метод, который вы использовали бы для решения этих уравнений: 3x+1=81,3x+1=81,3x+1=75,3x+1=75. Требует ли ваш метод логарифмирования обоих уравнений? Почему или почему нет?

      355.

      В чем разница между уравнением экспоненциального роста и уравнением экспоненциального распада?

      Самопроверка

      ⓐ После выполнения упражнений используйте этот контрольный список, чтобы оценить свое мастерство в выполнении целей этого раздела.

      Добавить комментарий

      Ваш адрес email не будет опубликован. Обязательные поля помечены *